Формула силы электрического тока: Определение силы тока. Единицы измерения силы тока

{2}-A v t\right) \rightarrow C_{2}=\frac{\varepsilon \varepsilon_{0}(A v t)}{d}(2.4)$$

где $\varepsilon_{0}$ – электрическая постоянная, переменной величиной при погружении системы в керосин является площадь обкладок S:

$$S_{2}=A \cdot v \cdot t ; S_{1}=A \cdot(A-v t)$$

Из выражений (2.4), (2.5) и условий задачи имеем:

$$d C=d C_{1}+d C_{2}=\frac{\varepsilon \varepsilon_{0} A v d t}{d}-\frac{\varepsilon_{0}}{d} A v d t(2.6)$$

Тогда подставив dC в формулу для силы тока (2.1) получаем:

$$I=U\left(\frac{\varepsilon \varepsilon_{0} A v}{d}-\frac{\varepsilon_{0}}{d} A v\right)=\frac{\varepsilon_{0} U A v}{d}(\varepsilon-1)$$

Ответ. $I=\frac{\varepsilon_{0} U A v}{d}(\varepsilon-1)$

Читать дальше: Формула силы.

Содержание

Формула силы тока в физике

Содержание:

Определение и формула силы тока

Определение

Электрическим током называют упорядоченное движение носителей зарядов. В металлах таковыми являются электроны, отрицательно заряженные частицы с зарядом, равным элементарному заряду. Направлением тока считают направление движения положительно заряженных частиц.

Силой тока (током) через некоторую поверхность S называют скалярную физическую величину, которую обозначают I, равную:

$$I=\frac{d q}{d t}$ (1)$

где q – заряд, проходящий сквозь поверхность S, t – время прохождения заряда. Выражение (1) определяет величину силы тока в момент времени t (мгновенное значение величины силы тока).

Некоторые виды силы тока

Ток носит название постоянного, если его сила и направление с течением времени не изменяются, тогда:

$$I=\frac{q}{t}(2)$$

Формула (2) показывает, что сила постоянного тока равна заряду, который проходит сквозь поверхность S в единицу времени.

Если ток является переменным, то выделяют мгновенную силу тока (1), амплитудную силу тока и эффективную силу тока. Эффективной величиной силы переменного тока (Ieff) называют такую силу постоянного тока, которая выполнит работу равную работе переменного тока в течение одного периода (T):

$$I_{e f f}=\sqrt{\frac{1}{T} \int_{0}^{T} I^{2} d t}(3)$$

Если переменный ток можно представить как синусоидальный:

$$I=I_{m} \sin \omega t$$

то Im – амплитуда силы тока ($\omega$ – частота силы переменного тока). {2}-A v t\right) \rightarrow C_{2}=\frac{\varepsilon \varepsilon_{0}(A v t)}{d}(2.4)$$

где $\varepsilon_{0}$ – электрическая постоянная, переменной величиной при погружении системы в керосин является площадь обкладок S:

$$S_{2}=A \cdot v \cdot t ; S_{1}=A \cdot(A-v t)$$

Из выражений (2.4), (2.5) и условий задачи имеем:

$$d C=d C_{1}+d C_{2}=\frac{\varepsilon \varepsilon_{0} A v d t}{d}-\frac{\varepsilon_{0}}{d} A v d t(2.6)$$

Тогда подставив dC в формулу для силы тока (2.1) получаем:

$$I=U\left(\frac{\varepsilon \varepsilon_{0} A v}{d}-\frac{\varepsilon_{0}}{d} A v\right)=\frac{\varepsilon_{0} U A v}{d}(\varepsilon-1)$$

Ответ. $I=\frac{\varepsilon_{0} U A v}{d}(\varepsilon-1)$

Читать дальше: Формула силы.

Формула электрического тока, по какой формуле можно найти, вычислить силу тока.

 

 

 

Тема: как рассчитать силу тока, зная напряжение и сопротивления по закону Ома.

 

Электрический ток, это именно та сила, которая течет во всей электротехники заставляя ее работать. Но сводить все к простому течению электротока по электрическим цепям в схемах неразумно, должна быть какая-то мера, определенная величина этой силы тока. Ведь если в электрической схеме пойдет слишком большой ток по проводникам, которые на него не рассчитаны, то просто эта схема выгорит. Из школьных уроков мы помним, что существуют так называемые формулы, которые и позволяют вычислять конкретные неизвестные величины имея при этом известные.

 

Вот самая базовая, наиболее используемая формула тока, по которой и вычисляется эта самая сила тока. В ней всего лишь три электрических величины (базовые электрические величины) — ток, напряжение и сопротивление.

 

 

Итак, сила тока на схемах обычно обозначается большой английской буквой «I». Единицей измерения тока является «Ампер». Формула тока звучит следующим образом — электрический ток равен отношению напряжения (разности потенциалов) к сопротивлению. То есть, чтобы найти силу тока нам нужно просто напряжение разделить на сопротивление. Единицей измерения электрического напряжения является «Вольт», а сопротивления «Ом». Следовательно, известные вольты делим на известные омы и получаем ранее неизвестные амперы.

 

 

 

 

Эта же формула еще называется законом Ома. Она помогает найти из двух известных величин третью, которая неизвестна. Чтобы найти напряжение, то нужно силу тока перемножить на сопротивление, а для нахождения сопротивления нужно будет напряжение разделить на силу тока. Все достаточно просто. Данная формула тока подходит и для постоянного тока и для переменного, но именно с активным сопротивлением. То есть, по ней можно рассчитать те электрические цепи (участки цепей в схемах), которые содержать сопротивления в виде обычных нагревателей, резисторов, лампочек (не имеющих индуктивную и емкостную составляющую). Индуктивностью обладают все катушки, а емкостью обладают все конденсаторы (они уже имеют реактивное сопротивление и рассчитываются по другой формуле).

 

Если говорить о формуле тока, которая ближе к научной сфере, то она уже будет иметь вид немного другой. Электрический ток изначально выражается как отношение количества электрических зарядов ко времени их прохождения через проводник.

 

 

Электрический ток это упорядоченное движение электрических зарядов (в твердых телах это электроны, а в жидких и газообразных телах это ионы). Так вот ток, это непосредственное движение этих зарядов и, естественно, что он определяется их количеством и временем течения. Электрические заряды измеряются в «Кулонах», ну а время в «секундах». Следовательно, чтобы узнать силу электрического тока нужно количество зарядов разделить на время их прохождения. То есть, кулоны делим на секунды и получаем амперы.

 

Повторюсь, что на практике при измерении и вычислении силы тока пользуются именно формулой закона Ома, поскольку приходится использовать при расчетах напряжение и сопротивление.

Именно они повсеместно будут встречаться в электрических схемах той или иной электротехники. Никаких кулонов (количества зарядов) вы при своей работе электриком не увидите!

 

 

Ну, и поскольку выше я затронул тему реактивного сопротивления, то пожалуй приведу формулу для нахождения силы тока именно для цепей, содержащих индуктивное и емкостное сопротивление.

 

 

По данной формуле можно найти силу тока, которая будет течь в электрической цепи с переменным, синусоидальным напряжением и содержащая реактивное сопротивление в виде катушки (индуктивности) или конденсатора (емкости). Думаю вы заметили, что в приведенной формуле изменился лишь тип сопротивления. Сама же основа — это все та же формула закона Ома, что была приведена в самом начале. Просто тут для нахождения индуктивного и емкостного сопротивления уже используются такие величины как частота, емкость и индуктивность, ну и еще «ПИ», которое равно 3,14.

 

P.S. Формулу электрического тока вы просто обязаны знать наизусть (если вы конечно электрик или электронщик). Формула закона Ома будет вам полезна очень много раз. Как только нужно найти силу тока, напряжение или сопротивление (зная любые две величины из трех) вы быстро и без проблем сразу подставляете числа в эту формулу и вычислите неизвестные электрические величины.

 

 

Сила тока: природа, формула, измерение амперметром

 

Наверное, каждый хотя бы раз в жизни ощущал на себе действие тока. Обыкновенная батарейка едва ощутимо пощипывает, если приложить ее к языку. Ток в квартирной розетке довольно сильно бьет, если коснуться оголенных проводов. А вот электрический стул и линии электропередач могут лишить жизни.

Во всех случаях мы говорим о действии электрического тока. Чем же так отличается один ток от другого, что разница в его воздействии столь существенна? Очевидно, есть некая количественная характеристика, которой можно объяснить такое различие. Ток, как известно, это передвигающиеся по проводнику электроны. Можно предположить, что чем больше через сечение проводника пробежит электронов, тем большее действие произведет ток.

Формула силы тока

Для того, чтобы охарактеризовать заряд, проходящий через проводник, ввели физическую величину, называемую силой электрического тока. Сила тока в проводнике – это количество электричества, проходящего через поперечное сечение проводника за единицу времени. Сила тока равна отношению электрического заряда ко времени его прохождения. Для расчета силы тока применяют формулу:

I=q/t,

где I- сила тока,
q — электрический заряд,
t — время.

За единицу силы тока в цепи принят 1 Ампер (1 А) в честь французского ученого Андре Ампера. На практике часто применяют кратные единицы: миллиамперы, микроамперы и килоамперы.

Измерение силы тока амперметром

Для измерения силы тока применяют амперметры. Амперметры бывают различными в зависимости от того, для каких измерений они рассчитаны. Соответственно, шкалу прибора градуируют в требуемых величинах. Амперметр подключается в любом месте сети последовательно. Место подключения амперметра не имеет значения, так как количество электричества, проходящее через цепь, в любом месте будет одинаково. Электроны не могут скапливаться в каких-либо местах цепи, они текут равномерно по всем проводам и элементам. При подключении амперметра до и после нагрузки он покажет одинаковые значения.

Первые ученые, исследовавшие электричество, не имели приборов дл измерения силы тока и величины заряда. Они проверяли наличие тока собственными ощущениями, пропуская его через свое тело. Довольно неприятный способ. На то время силы токов, с которыми они работали, были не очень велики, поэтому большинство исследователей отделывались лишь неприятными ощущениями. Однако, в наше время даже в быту, не говоря уже про промышленность, используются токи очень больших значений.

Следует знать, что для человеческого организма безопасной признана величина силы тока до 1 мА. Величина тока больше 100 мА может привести к серьезным повреждениям организма. Величина тока в несколько ампер может убить человека. При этом еще нужно учитывать индивидуальную восприимчивость организма, которая различна у каждого человека. Поэтому следует помнить о главном требовании при эксплуатации электроприборов – безопасность.

Нужна помощь в учебе?



Предыдущая тема: Ток в металлах: действия тока и направление тока
Следующая тема:&nbsp&nbsp&nbspЭлектрическое напряжение: определение, формула, вольтметр

Что такое сила тока — пояснения и формулы

Движение заряженных частиц в проводнике в электротехнике называется электрическим током. Электроток не характеризуется только прошедшим через проводник значением количества электрической энергии, так как за 60 минут через него может пройти электричество равное 1 Кулону, но и такое же количество электричества можно пропустить через проводник за одну секунду.

Что такое сила тока

Когда рассматривается количество электричества, протекающее через проводник за разные интервалы времени, понятно, что за меньший промежуток времени ток течет интенсивней, поэтому в характеристику электротока вводится еще одно определение — это сила тока, которая характеризуется протекающим в проводнике током за секунду времени. Единицей измерения величины силы проходящего тока в электротехнике принят ампер.

Иными словами, сила электрического тока в проводнике — это количество электричества, которое прошло через его сечение за секунду времени, маркировка литерой I. Силу тока измеряют в амперах — это единица измерения, которая равняется силе неизменяющегося тока, проходящего по бесконечным параллельным проводам с наименьшим круговым сечением, удаленным друг от друга на 100 см и расположенным в вакууме, который вызывает взаимодействие на метре длины проводника силой = 2*10 минус 7 степени Ньютона на каждые 100 см длины.

Специалисты часто определяют величину проходящего тока, на Украине (сила струму) она равна 1 амперу, когда через сечение проводника проходит каждую секунду 1 кулон электричества.

Формула определения силы тока:

Формула определения силы тока

В электротехнике можно увидеть частое применение других величин в определении значения силы проходящего тока: 1 миллиампер, который равен единица/ Ампер, 10 в минус третьей степени Ампер, один микроампер — это десять в минус шестой степени Ампер.

Зная количество электричества, прошедшее через проводник за определенный промежуток времени, можно вычислить силу тока (как говорят на Украине — силу струму) по формуле:

Формула силы тока

Когда электрическая цепь замкнута и не имеет ответвлений, тогда в каждом месте ее поперечного сечения протекает за секунду одинаковое количество электричества. Теоретически это объясняется невозможностью накапливания электрических зарядов в каком либо месте цепи, по этой причине сила тока везде одинакова.

Правило постоянства электрического тока в замкнутой цепи

Данное правило справедливо и в сложных цепях, когда есть ответвления, но относится к некоторым участкам сложной цепи, которые можно рассматривать в виде простой электроцепи.

Как измеряется сила тока

Величину силы тока измеряют прибором, который называется амперметр, а также для небольших значений — миллиамперметр и микроамперметр, который можно увидеть на фото внизу:

Амперметр Основы

Среди людей бытует мнение, что когда измеряется сила тока в проводнике до нагрузки (потребителя), то значение будет выше, чем после нее. Это ошибочное мнение, основанное на том, что якобы какое-то значение силы будет расходоваться на то, чтобы привести потребитель в действие. Электроток в проводнике — это процесс электромагнитный, в котором участвуют заряженные электроны, они направленно двигаются, но энергию передают не электроны, а электромагнитное поле, которое окружает проводник.

Количество электронов, вышедших из начала цепи, будет равно количеству электронов и после потребителя в конце цепи, они не могут быть израсходованы.

Измерение силы тока

Какие проводники бывают? Специалисты дают определение понятию «проводник» — это материал, в котором частицы, имеющие заряд, могут перемещаться свободно. Такие свойства на практике имеют почти все металлы, кислота и солевой раствор. А материал или вещество, в котором движение заряженных частиц затруднено или вообще невозможно, называются изоляторами (диэлектриками). Часто встречающиеся материалы-диэлектрики — это кварц или эбонит, искусственный изолятор.

Вывод

На практике современное оборудование работает с большими величинами тока, до сотни, а то и тысячи ампер, а также и с малыми значениями. Примером в повседневной жизни величины тока в разных приборах может быть электрическая плита, где она достигает значения в 5 А, а простая лампа накаливания может иметь величину 0,4 А, в фотоэлементе величина проходящего тока измеряется в микроамперах. В линиях городского общественного транспорта (троллейбус, трамвай) значение проходящего тока достигает 1000 А.

Похожие статьи:

Постоянный электрический ток. Направление тока, формула

 

Автор статьи — профессиональный репетитор, автор учебных пособий для подготовки к ЕГЭ Игорь Вячеславович Яковлев

Темы кодификатора ЕГЭ: постоянный электрический ток, сила тока, напряжение.

Электрический ток обеспечивает комфортом жизнь современного человека. Технологические достижения цивилизации — энергетика, транспорт, радио, телевидение, компьютеры, мобильная связь — основаны на использовании электрического тока.

Электрический ток — это направленное движение заряженных частиц, при котором происходит перенос заряда из одних областей пространства в другие.

Электрический ток может возникать в самых различных средах: твёрдых телах, жидкостях, газах. Порой и среды никакой не нужно — ток может существовать даже в вакууме! Мы поговорим об этом в своё время, а пока приведём лишь некоторые примеры.

• Замкнём полюса батарейки металлическим проводом. Свободные электроны провода начнут направленное движение от «минуса» батарейки к «плюсу».
Это — пример тока в металлах.

• Бросим в стакан воды щепотку поваренной соли . Молекулы соли диссоциируют на ионы, так что в растворе появятся свободные заряды: положительные ионы и отрицательные ионы . Теперь засунем в воду два электрода, соединённые с полюсами батарейки. Ионы начнут направленное движение к отрицательному электроду, а ионы — к положительному.
Это — пример прохождения тока через раствор электролита.

• Грозовые тучи создают столь мощные электрические поля, что оказывается возможным пробой воздушного промежутка длиной в несколько километров. В результате сквозь воздух проходит гигантский разряд — молния.
Это — пример электрического тока в газе.

Во всех трёх рассмотренных примерах электрический ток обусловлен движением заряженных частиц внутри тела и называется током проводимости.

• Вот несколько иной пример. Будем перемещать в пространстве заряженное тело. Такая ситуация согласуется с определением тока! Направленное движение зарядов — есть, перенос заряда в пространстве — присутствует. Ток, созданный движением макроскопического заряженного тела, называется конвекционным.

Заметим, что не всякое движение заряженных частиц образует ток. Например, хаотическое тепловое движение зарядов проводника — не направленное (оно совершается в каких угодно направлениях), и потому током не является (при возникновении тока свободные заряды продолжают совершать тепловое движение! Просто в этом случае к хаотическим перемещениям заряженных частиц добавляется их упорядоченный дрейф в определённом
направлении).
Не будет током и поступательное движение электрически нейтрального тела: хотя заряженные частицы в его атомах и совершают направленное движение, не происходит переноса заряда из одних участков пространства в другие.

 

Направление электрического тока

 

Направление движения заряженных частиц, образующих ток, зависит от знака их заряда. Положительно заряженные частицы будут двигаться от «плюса» к «минусу», а отрицательно заряженные — наоборот, от «минуса» к «плюсу». В электролитах и газах, например, присутствуют как положительные, так и отрицательные свободные заряды, и ток создаётся их встречным движением в обоих направлениях. Какое же из этих направлений принять за направление электрического тока?

Направлением тока принято считать направление движения положительных зарядов.

Попросту говоря, по соглашению ток течёт от «плюса» к «минусу» (рис. 1; положительная клемма источника тока изображена длинной чертой, отрицательная клемма — короткой).

Рис. 1. Направление тока

Данное соглашение вступает в некоторое противоречие с наиболее распространённым случаем металлических проводников. В металле носителями заряда являются свободные электроны, и двигаются они от «минуса» к «плюсу». Но в соответствии с соглашением мы вынуждены считать, что направление тока в металлическом проводнике противоположно движению свободных электронов. Это, конечно, не очень удобно.

Тут, однако, ничего не поделаешь — придётся принять эту ситуацию как данность. Так уж исторически сложилось. Выбор направления тока был предложен Ампером (договорённость о направлении тока понадобилась Амперу для того, чтобы дать чёткое правило определения направления силы, действующей на проводник с током в магнитном поле. Сегодня эту силу мы называем силой Ампера, направление которой определяется по правилу левой руки) в первой половине XIX века, за 70 лет до открытия электрона. К этому выбору все привыкли, и когда в 1916 году выяснилось, что ток в металлах вызван движением свободных электронов, ничего менять уже не стали.

 

Действия электрического тока

 

Как мы можем определить, протекает электрический ток или нет? О возникновении электрического тока можно судить по следующим его проявлениям.

1. Тепловое действие тока. Электрический ток вызывает нагревание вещества, в котором он протекает. Именно так нагреваются спирали нагревательных приборов и ламп накаливания. Именно поэтому мы видим молнию. В основе действия тепловых амперметров лежит тепловое расширение проводника с током, приводящее к перемещению стрелки прибора.

2. Магнитное действие тока. Электрический ток создаёт магнитное поле: стрелка компаса, расположенная рядом с проводом, при включении тока поворачивается перпендикулярно проводу. Магнитное поле тока можно многократно усилить, если обмотать провод вокруг железного стержня — получится электромагнит. На этом принципе основано действие амперметров магнитоэлектрической системы: электромагнит поворачивается в поле постоянного магнита, в результате чего стрелка прибора перемещается по шкале.

3. Химическое действие тока. При прохождении тока через электролиты можно наблюдать изменение химического состава вещества. Так, в растворе положительные ионы двигаются к отрицательному электроду, и этот электрод покрывается медью.

Электрический ток называется постоянным, если за равные промежутки времени через поперечное сечение проводника проходит одинаковый заряд.

Постоянный ток наиболее прост для изучения. С него мы и начинаем.

 

Сила и плотность тока

 

Количественной характеристикой электрического тока является сила тока. В случае постоянного тока абсолютная величина силы тока есть отношение абсолютной величины заряда , прошедшего через поперечное сечение проводника за время , к этому самому времени:

(1)

Измеряется сила тока в амперах (A). При силе тока в А через поперечное сечение проводника за с проходит заряд в Кл.

Подчеркнём, что формула (1) определяет абсолютную величину, или модуль силы тока.
Сила тока может иметь ещё и знак! Этот знак не связан со знаком зарядов, образующих ток, и выбирается из иных соображений. А именно, в ряде ситуаций (например, если заранее не ясно, куда потечёт ток) удобно зафиксировать некоторое направление обхода цепи (скажем, против часовой стрелки) и считать силу тока положительной, если направление тока совпадает с направлением обхода, и отрицательной, если ток течёт против направления обхода (сравните с тригонометрическим кругом: углы считаются положительными, если отсчитываются против часовой стрелки, и отрицательными, если по часовой стрелке).

В случае постоянного тока сила тока есть величина постоянная. Она показывает, какой заряд проходит через поперечное сечение проводника за с.

Часто бывает удобно не связываться с площадью поперечного сечения и ввести величину плотности тока:

(2)

где — сила тока, — площадь поперечного сечения проводника (разумеется, это сечение перпендикулярно направлению тока). С учётом формулы (1) имеем также:

Плотность тока показывает, какой заряд проходит за единицу времени через единицу площади поперечного сечения проводника. Согласно формуле (2), плотность тока измеряется в А/м2.

 

Скорость направленного движения зарядов

 

Когда мы включаем в комнате свет, нам кажется, что лампочка загорается мгновенно. Скорость распространения тока по проводам очень велика: она близка к км/с (скорости света в вакууме). Если бы лампочка находилась на Луне, она зажглась бы через секунду с небольшим.

Однако не следует думать, что с такой грандиозной скоростью двигаются свободные заряды, образующие ток. Оказывается, их скорость составляет всего-навсего доли миллиметра в секунду.

Почему же ток распространяется по проводам так быстро? Дело в том, что свободные заряды взаимодействуют друг с другом и, находясь под действием электрического поля источника тока, при замыкании цепи приходят в движение почти одновременно вдоль всего проводника. Скорость распространения тока есть скорость передачи электрического взаимодействия между свободными зарядами, и она близка к скорости света в вакууме. Скорость же, с которой сами заряды перемещаются внутри проводника, может быть на много порядков меньше.

Итак, подчеркнём ещё раз, что мы различаем две скорости.

1. Скорость распространения тока. Это — скорость передачи электрического сигнала по цепи. Близка к км/с.

2. Скорость направленного движения свободных зарядов. Это — средняя скорость перемещения зарядов, образующих ток. Называется ещё скоростью дрейфа.

Мы сейчас выведем формулу, выражающую силу тока через скорость направленного движения зарядов проводника.

Пусть проводник имеет площадь поперечного сечения (рис. 2). Свободные заряды проводника будем считать положительными; величину свободного заряда обозначим (в наиболее важном для практики случая металлического проводника это есть заряд электрона). Концентрация свободных зарядов (т. е. их число в единице объёма) равна .

Рис. 2. К выводу формулы

Какой заряд пройдёт через поперечное сечение нашего проводника за время ?

С одной стороны, разумеется,

(3)

С другой стороны, сечение пересекут все те свободные заряды, которые спустя время окажутся внутри цилиндра с высотой . Их число равно:

Следовательно, их общий заряд будет равен:

(4)

Приравнивая правые части формул (3) и (4) и сокращая на , получим:

(5)

Соответственно, плотность тока оказывается равна:

Давайте в качестве примера посчитаем, какова скорость движения свободных электронов в медном проводе при силе тока A.

Заряд электрона известен: Кл.

Чему равна концентрация свободных электронов? Она совпадает с концентрацией атомов меди, поскольку от каждого атома отщепляется по одному валентному электрону. Ну а концентрацию атомов мы находить умеем:

м

Положим мм . Из формулы (5) получим:

м/с.

Это порядка одной десятой миллиметра в секунду.

 

Стационарное электрическое поле

 

Мы всё время говорим о направленном движении зарядов, но ещё не касались вопроса о том, почему свободные заряды совершают такое движение. Почему, собственно, возникает электрический ток?

Для упорядоченного перемещения зарядов внутри проводника необходима сила, действующая на заряды в определённом направлении. Откуда берётся эта сила? Со стороны электрического поля!

Чтобы в проводнике протекал постоянный ток, внутри проводника должно существовать стационарное (то есть — постоянное, не зависящее от времени) электрическое поле. Иными словами, между концами проводника нужно поддерживать постоянную разность потенциалов.

Стационарное электрическое поле должно создаваться зарядами проводников, входящих в электрическую цепь. Однако заряженные проводники сами по себе не смогут обеспечить протекание постоянного тока.

Рассмотрим, к примеру, два проводящих шара, заряженных разноимённо. Соединим их проводом. Между концами провода возникнет разность потенциалов, а внутри провода — электрическое поле. По проводу потечёт ток. Но по мере прохождения тока разность потенциалов между шарами будет уменьшаться, вслед за ней станет убывать и напряжённость поля в проводе. В конце концов потенциалы шаров станут равны друг другу, поле в проводе обратится в нуль, и ток исчезнет. Мы оказались в электростатике: шары плюс провод образуют единый проводник, в каждой точке которого потенциал принимает одно и то же значение; напряжённость
поля внутри проводника равна нулю, никакого тока нет.

То, что электростатическое поле само по себе не годится на роль стационарного поля, создающего ток, ясно и из более общих соображений. Ведь электростатическое поле потенциально, его работа при перемещении заряда по замкнутому пути равна нулю. Следовательно, оно не может вызывать циркулирование зарядов по замкнутой электрической цепи — для этого требуется совершать ненулевую работу.

Кто же будет совершать эту ненулевую работу? Кто будет поддерживать в цепи разность потенциалов и обеспечивать стационарное электрическое поле, создающее ток в проводниках?

Ответ — источник тока, важнейший элемент электрической цепи.

Чтобы в проводнике протекал постоянный ток, концы проводника должны быть присоединены к клеммам источника тока (батарейки, аккумулятора и т. д.).

Клеммы источника — это заряженные проводники. Если цепь замкнута, то заряды с клемм перемещаются по цепи — как в рассмотренном выше примере с шарами. Но теперь разность потенциалов между клеммами не уменьшается: источник тока непрерывно восполняет заряды на клеммах, поддерживая разность потенциалов между концами цепи на неизменном уровне.

В этом и состоит предназначение источника постоянного тока. Внутри него протекают процессы неэлектрического (чаще всего — химического) происхождения, которые обеспечивают непрерывное разделение зарядов. Эти заряды поставляются на клеммы источника в необходимом количестве.

Количественную характеристику неэлектрических процессов разделения зарядов внутри источника — так называемую ЭДС — мы изучим позже, в соответствующем листке.

А сейчас вернёмся к стационарному электрическому полю. Каким же образом оно возникает в проводниках цепи при наличии источника тока?

Заряженные клеммы источника создают на концах проводника электрическое поле. Свободные заряды проводника, находящиеся вблизи клемм, приходят в движение и действуют своим электрическим полем на соседние заряды. Со скоростью, близкой к скорости света, это взаимодействие передаётся вдоль всей цепи, и в цепи устанавливается постоянный электрический ток. Стабилизируется и электрическое поле, создаваемое движущимися зарядами.

Стационарное электрическое поле — это поле свободных зарядов проводника, совершающих направленное движение.

Стационарное электрическое поле не меняется со временем потому, что при постоянном токе не меняется картина распределения зарядов в проводнике: на место заряда, покинувшего данный участок проводника, в следующий момент времени поступает точно такой же заряд. По этой причине стационарное поле во многом (но не во всём) аналогично полю электростатическому.

А именно, справедливы следующие два утверждения, которые понадобятся нам в дальнейшем (их доказательство даётся в вузовском курсе физики).

1. Как и электростатическое поле, стационарное электрическое поле потенциально. Это позволяет говорить о разности потенциалов (т. е. напряжении) на любом участке цепи (именно эту разность потенциалов мы измеряем вольтметром).
Потенциальность, напомним, означает, что работа стационарного поля по перемещению заряда не зависит от формы траектории. Именно поэтому при параллельном соединении проводников напряжение на каждом из них одинаково: оно равно разности потенциалов стационарного поля между теми двумя точками, к которым подключены проводники.
2. В отличие от электростатического поля, стационарное поле движущихся зарядов проникает внутрь проводника (дело в том, что свободные заряды, участвуя в направленном движении, не успевают должным образом перестраиваться и принимать «электростатические» конфигурации).
Линии напряжённости стационарного поля внутри проводника параллельны его поверхности, как бы ни изгибался проводник. Поэтому, как и в однородном электростатическом поле, справедлива формула , где — напряжение на концах проводника, — напряжённость стационарного поля в проводнике, — длина проводника.

сопротивления через силу тока и напряжение

Электротехника как область науки, занимающаяся использованием электроэнергии, в том числе ее получением, распределением и учетом, оперирует значениями тока, напряжения, мощности и сопротивления. Это основные величины. Кроме этого, имеется множество других характеристик и понятий, но в рамках данной статьи будут рассматриваться именно эти основополагающие понятия.

Многообразие устройств электротехники

Электрический ток

Согласно определению, ток представляет собой упорядоченное движение заряженных частиц в среде. Такими частицами могут быть свободные электроны или ионы, частицы вещества, в которых число протонов в ядре не равно количеству электронов, то есть имеющие определенный заряд, положительный или отрицательный. Электроток может быть постоянный или переменный.

Электрическое напряжение

Электрическое напряжение – это разность потенциалов на противоположных участках цепи. Точное определение понятия подразумевает работу по переносу электрического заряда между участками цепи.

Сопротивление

Любой проводник в цепи препятствует прохождению через себя тока. Данная характеристика определяет такую физическую величину, как сопротивление. Исходя из величины сопротивления, все вещества относят к проводникам или изоляторам. Точная граница весьма расплывчата, поэтому при некоторых условиях некоторые вещества можно отнести как к изоляторам, так и к проводникам. Участок электросхемы может иметь элемент с определенным значением величины, который именуется резистор.

Резисторы различных типов

Мощность

Скорость преобразования, передачи и потребления электрической энергии определяется мощностью.

Взаимосвязь параметров электрической цепи

Все параметры любой электрической цепи строго взаимосвязаны, поэтому в любой момент времени можно точно определить величину любого из них, зная остальные.

К сведению. Основополагающий закон, по которому производится большинство расчетов, – закон Ома, согласно которому сила тока обратно пропорциональна его сопротивлению и прямо пропорциональна приложенной разности потенциалов.

Закон Ома и его основатель

Формула напряжения тока закона Ома выглядит следующим образом:

I=U/R.

Так, цепь с большим напряжением пропускает больший ток, а при одинаковом напряжении ампераж будет больше там, где меньше сопротивление.

Принятые обозначения в формуле расчета напряжения и тока понятны во всем мире:

  • I – сила тока;
  • U – напряжение;
  • R – сопротивление.

Путем простейшего математического преобразования находится формула расчета сопротивления через силу тока и напряжение.

Кроме закона Ома, используется формула расчета мощности:

P=U∙I.

Символом P здесь обозначена мощность тока.

Любая схема может содержать участки, где имеется последовательное соединение, или есть элемент, подключенный параллельно. Расчеты при этом усложняются, но базовые формулы остаются одинаковыми.

Единицы измерения в формуле

Невозможно выполнять расчеты или измерения, не зная, какими величинами оперировать. Общепринятые обозначения, согласно международной системе измерения СИ:

  • Напряжение – Вольт. Обозначается символом В или V в англоязычной литературе;
  • Сила тока – Ампер. Обозначается символом А;
  • Электрическое сопротивление – Ом. Используется обозначение Ом или Ohm;
  • Электрическая мощность – Ватт. Обозначается как Вт или W.

Как работает закон в реальной жизни

Используя совместно формулу расчета мощности и закон Ома, можно производить вычисления, не зная одной из величин. Самый простой пример – для лампы накаливания известны только ее мощность и напряжение. Применяя приведенные выше формулы, можно легко определить параметры нити накаливания и ток через нее.

Лампа накаливания

Сила тока формула через мощность:

I=P/U;

Сопротивление:

R=U/I.

Такой же результат можно найти из мощности, не прибегая к промежуточным расчетам:

R=U2/P.

Аналогично можно вычислить любую величину, зная только две из них. Для упрощения преобразований имеется мнемоническое отображение формул, позволяющее находить любые величины.

Правило для запоминания расчетов

Внимательно посмотрев на формулы, можно заметить, что, если уменьшить напряжение на лампе в два раза, ожидаемая мощность не снизится аналогично в два раза, а в четыре, согласно формуле:

P=U2/R.

Это довольно распространенная ошибка среди далеких от электротехники людей, которые неправильно соотносят мощность и напряжение, а также их действие на остальные параметры.

Кстати. Сила тока, найденная через сопротивление и напряжение, справедлива как для постоянного, так и для переменного тока, если в ней не используются такие элементы, как конденсатор или индуктивность.

Облегчить расчеты можно, используя онлайн калькулятор.

Пример с обычной водой

Существуют вещества, которые можно отнести одновременно к проводникам и изоляторам. Самый простой пример – обыкновенная вода. Дистиллированная вода является хорошим изолятором, но наличие в ней практически любых примесей делает ее проводником. Особенно это относится к солям различных металлов. При растворении в воде соли диссоциируются на ионы, их наличие – прямой повод для возникновения тока. Чем больше концентрация солей, тем меньшим сопротивлением будет обладать вода.

Зависимость сопротивления воды от содержания солей

Для наглядности можно взять дистиллированную воду для приготовления электролита для автомобильных аккумуляторных батарей.  Опустив щупы омметра в воду, можно увидеть, что его показания велики. Добавление всего нескольких кристаллов поваренной соли через некоторое время вызывает резкое уменьшение сопротивления, которое будет тем меньше, чем больше соли перейдет в раствор.

По какой формуле определяется напряжение

Использование той или иной формулы напряжения электрического тока для вычисления зависит от того, какие величины известны:

  • Ток и сопротивление – U=I∙R;
  • Ток и мощность – U=P/I;
  • Мощность и сопротивление – U=√P∙R

Различные используемые величины

Кроме основных величин: вольт, ампер, ом, ватт, используют кратные, большие или меньшие. Для обозначений применяют соответствующие приставки:

  • Кило – 1000;
  • Мега – 1000000;
  • Гига – 1000000000;
  • Милли – 0. 001.

Таким образом, получается:

  • Киловольт (кВ) – тысяча вольт;
  • Мегаватт (Мвт) – миллион ватт;
  • Миллиом (мОм) – одна тысячная Ом;
  • Гигаватт (ГВт) – тысяча мегаватт или миллиард ватт.

Как найти напряжение

Формула нахождения напряжения как разности потенциалов в электрическом поле:

U=ϕA-ϕB, где ϕAи ϕB – потенциалы в точках А и В, соответственно.

Также можно записать напряжение как работу по переносу единицы заряда из точки А в точку В в электрическом поле:

U=A/q, где q – величина заряда.

Работа тем больше, чем выше напряженность электрического поля Е, то есть сила, действующая на неподвижный заряд.

Потенциальную энергию заряда в электростатическом поле называют электростатический потенциал.

Гидравлическая аналогия

Чтобы легче усвоить законы электрических цепей, можно представить себе аналогию с гидравлической системой, в которой соединение насоса и трубопроводов образует замкнутую систему. Для этого нужны следующие соответствия:

  • Источник питания – насос;
  • Проводники – трубы;
  • Электроток – движение воды.

Без особых усилий становится понятнее, что чем меньше диаметр труб, тем медленнее по ним движется вода. Чем мощнее насос, тем большее количество воды он способен перекачать. При одинаковой мощности насоса уменьшение диаметра труб приведет к снижению потока воды.

Гидравлическая аналогия

Измерительные приборы

Для измерения параметров электрических цепей служат измерительные приборы:

  • Вольтметр;
  • Амперметр;
  • Омметр.

Наиболее часто используется класс комбинированных устройств, в которых переключателем выбирается измеряемая величина – ампервольтомметры или авометры.

Один из самых распространенных авометров

Типичные напряжения

Для стандартизации и возможности использования различного оборудования в быту и технике применяются электрические сети со стандартными значениями:

  • Бытовая сеть –220В;
  • Бортовая сеть автомобиля – 12 или 24В;
  • Батареи и аккумуляторы – 1. 5, 3 или 9В.

Потенциал Гальвани

В электрохимии используется понятие потенциала Гальвани, который означает разность потенциала между различными фазами вещества, например, между электродом и электролитом, между электродами из разнородных металлов.

Видео

Электрический ток — Веб-формулы

Электрический ток определяется по формуле:

I = В / R

Соответствующие единицы:
ампер (А) = вольт (В) / Ом (Ом)

Эта формула выводится из закона Ома. . Где у нас:
В: напряжение
I: ток
R: сопротивление

Если электрическая мощность и полное сопротивление известны, то ток можно определить по следующей формуле:

I = √ ( P / R )

Соответствующие единицы:
Ампер (А) = √ (Ватт (Вт) / Ом (Ом))

Где P — электрическая мощность.


Электрический ток
Скорость потока заряда через поперечное сечение некоторой области металлического провода (или электролита) называется током через эту область.

Если скорость потока заряда непостоянна, тогда ток в любой момент задается дифференциальным пределом: I = dQ / dt.

Если заряд Q протекает по цепи в течение времени t, то
I = Q / t.

Единица измерения тока S.I называется ампер (А) (кулон в секунду).
1 ампер = 6,25 × 10 8 электронов / сек

В металлических проводниках ток возникает из-за движения электронов, тогда как в электролитах и ​​ионизированных газах как электроны, так и положительные ионы движутся в противоположном направлении. Направление тока принимается за направление движения положительных зарядов.

В проводимости, хотя ток возникает только за счет электронов, ранее предполагалось, что ток возникает из-за положительных зарядов, протекающих от положительного полюса батареи к отрицательному.Поэтому направление тока считается противоположным потоку электронов.

Если ток постоянный: Δq = I.Δt

функция времени:

Заряд = Площадь под графиком = ½ × t 0 × I 0

To Найти ток в электрической цепи
Для простой цепи или одиночного провода мы имеем:

Для сложной цепи с более чем одним проводом мы можем определить ток с помощью двух законов Кирхгофа

Первый закон: Этот закон основан на принципе сохранения заряда и утверждает, что в электрической цепи (или сети проводов) алгебраическая сумма токов, встречающихся в точке, равна нулю.

Стрелка, отмеченная на схеме, представляет направление обычного тока, то есть направление потока положительного заряда, тогда как направление потока электронов дает направление электронного тока, противоположное направлению обычного тока.
I 1 + I 4 + I 5 = I 3 + I 2 + I 6

Второй закон: Алгебраическая сумма произведения тока и сопротивление в любом замкнутом контуре цепи равно алгебраической сумме электродвижущих сил, действующих в этом контуре.
Математически.

Электродвижущие силы — ЭДС () источника определяется как работа, совершаемая на единицу заряда при прохождении положительного заряда через гнездо ЭДС от конца с низким потенциалом к ​​концу с высоким потенциалом. Таким образом,
𝜖 = w / Q

Когда ток не течет, ЭДС источника в точности равна разности потенциалов между его концами. Единица ЭДС такая же, как и у потенциала, то есть вольт.

Средний поток электронов в проводнике, не подключенном к батарее, равен нулю, т.е. количество свободных электронов, пересекающих любой участок проводника слева направо, равно количеству электронов, пересекающих участок проводника справа налево. ток не течет по проводнику, пока он не будет подключен к батарее.

Скорость дрейфа свободных электронов в металлическом проводнике

В отсутствие электрического поля свободные электроны в металле беспорядочно вращаются во всех направлениях, поэтому их средняя скорость равна нулю.При приложении электрического поля они ускоряются в направлении, противоположном направлению поля, и поэтому имеют общий дрейф в этом направлении. Однако из-за частых столкновений с атомами их средняя скорость очень мала. Эта средняя скорость, с которой электроны движутся в проводнике под действием разности потенциалов, называется дрейфовой скоростью .

Если E — приложенное поле, e — заряд электрона, m — масса электрона и τ — временной интервал между последовательными столкновениями (время релаксации), то ускорение электрона составляет

Поскольку средняя скорость сразу после столкновения равна нулю, а непосредственно перед следующим столкновением это τ, скорость дрейфа должна быть:

Если I — ток через проводник и n количество свободных электронов на единицу объема, тогда можно показать, что:

Подвижность µ носителя заряда определяется как скорость дрейфа на единицу электрического поля:

Плотность тока (J)
(i)
(ii) S.I Единица J = Am -2 .
(iii) Плотность тока — это векторная величина, ее направление — это направление потока положительного заряда в данной точке внутри проводника.
(iv) Размеры плотности тока = [M 0 L -2 T o A 1 ]

Носители тока: заряженные частицы, поток которых в определенном направлении составляет электрический ток, являются носителями тока. . Носители тока могут иметь положительный или отрицательный заряд.Ток переносится электронами в проводниках, ионами в электролитах, электронами и дырками в полупроводниках.

Пример 1: Частица с зарядом q кулонов описывает круговую орбиту. Если радиус орбиты равен R, а частота орбитального движения частиц равна f, то найти ток на орбите.

Решение: Через любой участок орбиты заряд проходит f раз за одну секунду. Следовательно, через этот участок общий заряд, проходящий за одну секунду, равен fq.По определению i = fq.

Пример 2: Ток в проводе изменяется со временем в соответствии с уравнением I = 4 + 2t, где I — в амперах, а t — в секундах. Вычислите количество заряда, прошедшего через поперечное сечение провода за время от t = 2 с до t = 6 с.

Решение: Пусть dq будет изменением, которое произошло за небольшой интервал времени dt.
Тогда dq = I dt = (4 + 2t) dt

Следовательно, полный заряд, прошедший за интервал t = 2 с и t = 6, равен
q = ∫ 6 2 (4 + 2t) dt = 48 кулонов

Пример 3: Дан токоведущий провод неоднородного сечения.Что из следующего является постоянным по всей сети?
(a) Только ток
(b) Ток и скорость дрейфа
(c) Только скорость дрейфа
(d) Ток, скорость дрейфа

Решение : (a)

Пример4 : Когда разность потенциалов на данном медном проводе увеличивается, скорость дрейфа составляет
носители заряда:
(а) Уменьшается
(б) Увеличивается
(в) Остается прежним
(г) Уменьшается до нуля
Решение : (б)

Электрический Current: Определение, Единица, Формула, Типы (с примерами)

Обновлено 28 декабря 2020 г.

Автор: Кевин Бек

Электрический заряд: Какую автоматическую реакцию вызывает эта фраза, когда вы ее читаете? Может быть, ощущение покалывания или образ молнии, раскалывающего небо? Красочный дисплей мигающих огней в таком городе, как Париж или Лас-Вегас? Возможно, даже насекомое, которое каким-то образом светится в темноте, пробираясь через ваш лагерь?

До недавних столетий ученые не только не имели возможности измерить скорость света, но и не знали, какие физические явления лежат в основе того, что сейчас известно как «электричество».В 1800-х годах физики впервые узнали о мельчайших частицах, участвующих в потоке тока (свободные электроны), а также о природе сил, заставляющих их двигаться. Было ясно, что электричество может принести значительную пользу, если оно может быть безопасно «изготовлено» или «захвачено», а электрическая энергия используется для выполнения работы.

Поток электрического заряда легко возникает в веществах, классифицируемых как проводящие материалы , тогда как в изоляторах ему препятствуют.В металлическом проводе, таком как медный провод, например, можно создать разность потенциалов на концах провода, вызывая поток заряда и создавая ток.

Определение электрического тока

Электрический ток — это средняя скорость потока электрического заряда (то есть заряда в единицу времени) через точку в пространстве. Этот заряд переносится электронами , движущимися по проводу в электрической цепи. Чем больше электронов проходит через эту точку за секунду, тем больше величина тока.

Единицей измерения тока в системе СИ является ампер (А), часто неофициально называемый «ампер». Сам электрический заряд измеряется в кулонах (Кл).

  • Заряд одного электрона равен -1,60 × 10 -19 Кл, а заряд протона равен по величине, но положительный знак . Это число считается фундаментальным зарядом e . Таким образом, основная единица ампер — кулоны в секунду (Кл / с).

По соглашению, электрический ток течет в направлении, противоположном потоку электронов .Это связано с тем, что направление тока было описано до того, как ученые узнали, какие носители заряда двигались под действием электрического поля. Для всех практических целей положительные заряды, движущиеся в положительном направлении, дают тот же физический (вычислительный) результат, что и отрицательные заряды, движущиеся в отрицательном направлении, когда дело касается электрического тока.

Электроны движутся к положительному выводу в электрической цепи. Таким образом, поток электронов или движущийся заряд находится далеко от отрицательного вывода.Движение электронов в медной проволоке или другом проводящем материале также создает магнитное поле , направление и величина которого определяются направлением электрического тока и, следовательно, движением электронов; это принцип, на котором построен электромагнит .

Формула электрического тока

Для базового сценария обычного тока заряда, движущегося по проводу, формула для тока имеет следующий вид:

I = neAv_d

, где n — количество зарядов на кубический метр (m 3 ), e — основной заряд, A — площадь поперечного сечения провода, а v d — дрейф скорость .

Хотя ток имеет как величину, так и направление, это скалярная величина, а не векторная величина, поскольку она не подчиняется законам сложения векторов.

Формула закона Ома

Закон Ома дает формулу для определения тока, который будет протекать через проводник:

I- \ frac {V} {R}

, где V напряжение , или разность электрических потенциалов , измеренная в вольтах, а R — электрическое сопротивление току, измеренное в Ом (Ом).

Подумайте о напряжении как о «тянущей силе» (хотя эта «электродвижущая сила» в буквальном смысле не является силой), специфичной для электрических зарядов. Когда противоположные заряды разделены, они притягиваются друг к другу таким образом, что уменьшается с увеличением расстояния между ними. Это примерно аналог гравитационной потенциальной энергии в классической механике; гравитация «хочет» падения высоких предметов на Землю, а напряжение «хочет», чтобы разделенные (противоположные) заряды столкнулись вместе.

Значение напряжения

Вольт эквивалентно джоулям на кулон, или Дж / Кл.Таким образом, у них есть единицы энергии на единицу заряда. Таким образом, ток, умноженный на напряжение, дает единицы (Кл / с) (Дж / Кл) = (Дж / с), что переводится в единицы (в данном случае электрической) мощности:

P = IV

Объединение этого с законом Ома дает переходят к другим полезным математическим соотношениям, связанным с протеканием тока: P = I 2 R и P = V 2 / R. Они показывают, среди прочего, что при фиксированном уровне тока мощность пропорциональна сопротивлению, тогда как при фиксированном напряжении мощность составляет обратно пропорционально сопротивлению .

В то время как движущиеся заряды (ток) индуцируют магнитное поле, магнитное поле само может вызывать напряжение в проводе.

Типы тока

  • Постоянный ток (DC): Это происходит, когда все электроны непрерывно текут в одном направлении. Это тип тока в цепи, подключенной к стандартной батарее. Батареи, конечно, могут поставлять и поставляют лишь исчезающе малое количество энергии, необходимой для питания человеческой цивилизации, хотя постоянно совершенствующиеся технологии в области солнечных элементов сулят более высокий потенциал для хранения энергии.
  • Переменный ток (AC): Здесь электроны колеблются взад и вперед (в некотором смысле «покачиваются») очень быстро. Этот тип тока часто легче генерировать на электростанции, и он также приводит к меньшим потерям энергии на большом расстоянии, поэтому он является стандартом, используемым сегодня. Все лампочки и другие электроприборы в стандартном доме начала 21 века питаются от сети переменного тока.

При переменном токе напряжение изменяется синусоидальным образом и задается в любой момент времени t выражением V = V 0 sin (2πft), где V 0 — начальное напряжение, а f — частота или количество полных циклов напряжения (от максимального к минимальному и обратно к максимальному значению) в каждую секунду.

Измерение тока

Амперметр — это устройство, которое используется для измерения тока путем последовательного, а не параллельного подключения в электрическую цепь. (Параллельная схема имеет несколько проводов между соединениями — другими словами, у источника питания, конденсаторов и резисторов — в цепи.) Принцип ее работы основан на том, что ток одинаков во всех частях провода между двумя соединениями.

Амперметр имеет известное низкое внутреннее сопротивление и настроен на полное отклонение (FSD) на при заданном уровне тока, часто равном 0.015 А или 15 мА. Если вы знаете напряжение и управляете сопротивлением с помощью функции шунтирующего сопротивления амперметра, вы можете определить ток; вы знаете, какое значение тока должно быть , используя закон Ома.

Примеры электрического тока

1. Вычислите скорость дрейфа электронов в цилиндрическом медном проводе радиусом 1 мм (0,001 м), по которому течет ток 15 А, учитывая, что для меди n = 8,342 × 10 28 э / м 3 .

I = neAv_d \ подразумевает v_d = \ frac {I} {neA}

Площадь A поперечного сечения провода равна πr 2 , или π (0.{-4} \ text {m / s}

  • Знак минус указывает на то, что направление противоположно направлению тока, как и ожидалось для электронов.

2. Найдите ток I в цепи на 120 В, в которой последовательно подключены резисторы 2 Ом, 4 Ом и 6 Ом.

Последовательные резисторы являются просто аддитивными (в параллельных цепях сумма общего сопротивления является суммой обратных величин отдельных значений сопротивления). Таким образом:

I = \ frac {V} {R} = \ frac {120} {2 + 4 + 6} = 10 \ text {A}

3.2 \ times 15 = 6000 \ text {W} \ text {и} V = IR = 20 \ times 15 = 300 \ text {V}

Учебное пособие по физике: Электрический ток

Если два требования электрической цепи выполнены, заряд будет проходить через внешнюю цепь. Говорят, что есть ток — поток заряда. Использование слова ток в этом контексте означает просто использовать его, чтобы сказать, что что-то происходит в проводах — заряд движется. Однако ток — это физическая величина, которую можно измерить и выразить численно.Как физическая величина, ток — это скорость, с которой заряд проходит через точку в цепи. Как показано на диаграмме ниже, ток в цепи можно определить, если можно измерить количество заряда Q , проходящего через поперечное сечение провода за время t . Ток — это просто соотношение количества заряда и времени.

Текущее — это величина ставки. В физике есть несколько скоростных величин. Например, скорость — это величина скорости — скорость, с которой объект меняет свое положение.Математически скорость — это отношение изменения положения к времени. Ускорение — это величина скорости — скорость, с которой объект меняет свою скорость. Математически ускорение — это отношение изменения скорости к времени. А мощность — это величина скорости — скорость, с которой работа выполняется на объекте. Математически мощность — это отношение работы к времени. В каждом случае величины скорости математическое уравнение включает некоторую величину во времени. Таким образом, ток как величина скорости будет математически выражен как

.

Обратите внимание, что в приведенном выше уравнении используется символ I для обозначения величины тока.

Как обычно, когда количество вводится в Классе физики, также вводится стандартная метрическая единица, используемая для выражения этой величины. Стандартная метрическая единица измерения силы тока — ампер . Ампер часто сокращается до А и обозначается условным обозначением A . Ток в 1 ампер означает, что 1 кулон заряда проходит через поперечное сечение провода каждую 1 секунду.

1 ампер = 1 кулон / 1 секунда

Чтобы проверить свое понимание, определите ток для следующих двух ситуаций.Обратите внимание, что в каждой ситуации дается некоторая посторонняя информация. Нажмите кнопку Проверить ответ , чтобы убедиться, что вы правы.

Провод изолируют поперечным сечением 2 мм и определяют, что заряд 20 C пройдет через него за 40 с.

Сечение провода длиной 1 мм изолируется, и определяется, что заряд 2 Кл проходит через него за 0,5 с.

I = _____ Ампер

I = _____ Ампер

Обычное направление тока

Частицы, которые переносят заряд по проводам в цепи, являются подвижными электронами.Направление электрического поля в цепи по определению является направлением, в котором проталкиваются положительные испытательные заряды. Таким образом, эти отрицательно заряженные электроны движутся в направлении, противоположном электрическому полю. Но в то время как электроны являются носителями заряда в металлических проводах, носителями заряда в других цепях могут быть положительные заряды, отрицательные заряды или и то, и другое. Фактически, носители заряда в полупроводниках, уличных фонарях и люминесцентных лампах одновременно являются как положительными, так и отрицательными зарядами, движущимися в противоположных направлениях.

Бен Франклин, проводивший обширные научные исследования статического и токового электричества, считал положительные заряды носителями заряда. Таким образом, раннее соглашение о направлении электрического тока было установлено в том направлении, в котором будут двигаться положительные заряды. Это соглашение прижилось и используется до сих пор. Направление электрического тока условно является направлением движения положительного заряда. Таким образом, ток во внешней цепи направлен от положительной клеммы к отрицательной клемме батареи.Электроны действительно будут двигаться по проводам в противоположном направлении. Зная, что настоящими носителями заряда в проводах являются отрицательно заряженные электроны, это соглашение может показаться немного странным и устаревшим. Тем не менее, это соглашение, которое используется во всем мире, и к которому студент-физик может легко привыкнуть.

Зависимость тока от скорости дрейфа

Ток связан с количеством кулонов заряда, которые проходят точку в цепи за единицу времени.Из-за своего определения его часто путают со скоростью дрейфа количества. Скорость дрейфа означает среднее расстояние, пройденное носителем заряда за единицу времени. Как и скорость любого объекта, скорость дрейфа электрона, движущегося по проводу, — это отношение расстояния ко времени. Путь типичного электрона через проволоку можно описать как довольно хаотический зигзагообразный путь, характеризующийся столкновениями с неподвижными атомами. Каждое столкновение приводит к изменению направления электрона.Однако из-за столкновений с атомами в твердой сети металлического проводника на каждые три шага вперед приходится два шага назад. С электрическим потенциалом, установленным на двух концах цепи, электрон продолжает движение вперед на . Прогресс всегда идет к положительной клемме. Однако общий эффект бесчисленных столкновений и высоких скоростей между столкновениями состоит в том, что общая скорость дрейфа электрона в цепи ненормально мала. Типичная скорость дрейфа может составлять 1 метр в час.Это медленно!

Тогда можно спросить: как может быть ток порядка 1 или 2 ампер в цепи, если скорость дрейфа составляет всего около 1 метра в час? Ответ таков: существует много-много носителей заряда, движущихся одновременно по всей длине цепи. Ток — это скорость, с которой заряд пересекает точку в цепи. Сильный ток является результатом нескольких кулонов заряда, пересекающих поперечное сечение провода в цепи. Если носители заряда плотно упакованы в провод, тогда не обязательно должна быть высокая скорость, чтобы иметь большой ток.То есть носители заряда не должны преодолевать большое расстояние за секунду, их просто должно быть много, проходящих через поперечное сечение. Ток не имеет отношения к тому, насколько далеко заряды перемещаются за секунду, а скорее к тому, сколько зарядов проходит через поперечное сечение провода в цепи.

Чтобы проиллюстрировать, насколько плотно упакованы носители заряда, мы рассмотрим типичный провод, который используется в цепях домашнего освещения — медный провод 14-го калибра. В срезе этой проволоки длиной 0,01 см (очень тонком) их будет целых 3.51 x 10 20 атомов меди. Каждый атом меди имеет 29 электронов; маловероятно, что даже 11 валентных электронов одновременно будут двигаться как носители заряда. Если мы предположим, что каждый атом меди вносит только один электрон, то на тонком 0,01-сантиметровом проводе будет целых 56 кулонов заряда. При таком большом количестве подвижного заряда в таком маленьком пространстве малая скорость дрейфа может привести к очень большому току.

Чтобы проиллюстрировать это различие между скоростью дрейфа и течением, рассмотрим аналогию с гонками.Предположим, что была очень большая гонка черепах с миллионами и миллионами черепах на очень широкой гоночной трассе. Черепахи не очень быстро двигаются — у них очень низкая скорость дрейф . Предположим, что гонка была довольно короткой — скажем, длиной 1 метр — и что значительный процент черепах достиг финишной черты в одно и то же время — через 30 минут после начала гонки. В таком случае течение будет очень большим — миллионы черепах пересекают точку за короткий промежуток времени.В этой аналогии скорость связана с тем, насколько далеко черепахи перемещаются за определенный промежуток времени; а ток зависит от того, сколько черепах пересекли финишную черту за определенный промежуток времени.

Природа потока заряда

Как только было установлено, что средняя скорость дрейфа электрона очень и очень мала, вскоре возникает вопрос: почему свет в комнате или в фонарике загорается сразу после включения переключателя? Разве не будет заметной задержки перед тем, как носитель заряда перейдет от переключателя к нити накала лампочки? Ответ — нет! и объяснение того, почему раскрывает значительную информацию о природе потока заряда в цепи.

Как упоминалось выше, носителями заряда в проводах электрических цепей являются электроны. Эти электроны просто поставляются атомами меди (или любого другого материала, из которого сделана проволока) внутри металлической проволоки. Как только переключатель переводится в положение на , цепь замыкается, и на двух концах внешней цепи устанавливается разность электрических потенциалов. Сигнал электрического поля распространяется почти со скоростью света ко всем подвижным электронам в цепи, приказывая им начать марш и марш .По получении сигнала электроны начинают двигаться по зигзагообразной траектории в обычном направлении. Таким образом, щелчок переключателя вызывает немедленную реакцию во всех частях схемы, заставляя носители заряда повсюду двигаться в одном и том же направлении. В то время как фактическое движение носителей заряда происходит с медленной скоростью, сигнал, который информирует о начале движения, движется со скоростью, составляющей долю от скорости света.

Электроны, которые зажигают лампочку в фонарике, не должны сначала пройти от переключателя через 10 см провода к нити накала.Скорее электроны, которые зажигают лампочку сразу после того, как переключатель повернут на на , являются электронами, которые присутствуют в самой нити. Когда переключатель повернут, все подвижные электроны повсюду начинают движение; и именно подвижные электроны, присутствующие в нити накала, непосредственно ответственны за зажигание ее колбы. Когда эти электроны покидают нить накала, в нее входят новые электроны, которые ответственны за зажигание лампы. Электроны движутся вместе, как вода в трубах дома.Когда кран поворачивается с на , вода в кране выходит из крана. Не нужно долго ждать, пока вода из точки входа в ваш дом пройдет по трубам к крану. Трубы уже заполнены водой, и вода во всем водном контуре одновременно приводится в движение.

Развиваемая здесь картина потока заряда представляет собой картину, на которой носители заряда подобны солдатам, идущим вместе, повсюду с одинаковой скоростью.Их движение начинается немедленно в ответ на установление электрического потенциала на двух концах цепи. В электрической цепи нет места, где носители заряда расходуются или расходуются. Хотя энергия, которой обладает заряд, может быть израсходована (или лучше сказать, что электрическая энергия преобразуется в другие формы энергии), сами носители заряда не распадаются, не исчезают или иным образом не удаляются из схема. И нет места в цепи, где бы носители заряда начали скапливаться или накапливаться.Скорость, с которой заряд входит во внешнюю цепь на одном конце, такая же, как скорость, с которой заряд выходит из внешней цепи на другом конце. Ток — скорость потока заряда — везде одинакова. Поток заряда подобен движению солдат, идущих вместе, повсюду с одинаковой скоростью.

Проверьте свое понимание

1.Говорят, что ток существует всякий раз, когда _____.

а. провод заряжен

г. аккумулятор присутствует

г. электрические заряды несбалансированные

г. электрические заряды движутся по петле

2. У тока есть направление. По соглашению ток идет в направлении ___.

а. + заряды перемещаются

г.- электроны движутся

г. + движение электронов

3. Скорость дрейфа подвижных носителей заряда в электрических цепях ____.

а. очень быстро; меньше, но очень близко к скорости света

г. быстрый; быстрее, чем самая быстрая машина, но далеко не скорость света

г. медленный; медленнее Майкла Джексона пробегает 220-метровую

г.очень медленно; медленнее улитки

4. Если бы электрическую цепь можно было сравнить с водяной цепью в аквапарке, то ток был бы аналогичен ____.

Выбор:

A. давление воды

млрд. Галлонов воды, стекающей с горки в минуту

С.вода

D. нижняя часть салазок

E. водяной насос

F. верх слайда

5. На схеме справа изображен токопроводящий провод. Две площади поперечного сечения расположены на расстоянии 50 см друг от друга. Каждые 2,0 секунды через каждую из этих областей проходит заряд 10 ° C.Сила тока в этом проводе ____ А.

а. 0,10

г. 0,25

г. 0,50

г. 1.0

e. 5,0

ф. 20

г. 10

ч.40

и. ни один из этих

6. Используйте диаграмму справа, чтобы заполнить следующие утверждения:

а. Ток в один ампер — это поток заряда со скоростью _______ кулонов в секунду.

г. Когда заряд 8 Кл проходит через любую точку цепи за 2 секунды, ток составляет ________ А.

г. Если за 10 секунд поток заряда проходит через точку A (диаграмма справа) на 5 ° C, то ток равен _________ A.

г. Если ток в точке D равен 2,0 А, то _______ C заряда проходит через точку D за 10 секунд.

e. Если 12 ° C заряда пройдет мимо точки A за 3 секунды, то 8 C заряда пройдут мимо точки E за ________ секунд.

ф. Верно или неверно:

Ток в точке E значительно меньше тока в точке A, поскольку в лампочках расходуется заряд.

Electric Current — The Physics Hypertextbook

Обсуждение

определений

текущий

Электрический ток определяется как скорость, с которой заряд проходит через поверхность (например, поперечное сечение провода). Несмотря на то, что оно относится ко многим различным вещам, слово ток часто используется само по себе вместо более длинного, более формального «электрического тока».Прилагательное «электрический» подразумевается контекстом описываемой ситуации. Фраза «ток через тостер», несомненно, относится к потоку электронов через нагревательный элемент, а не к потоку ломтиков хлеба через прорези.

Как и все величины, определяемые как скорость, есть два способа записать определение электрического тока — средний ток для тех, кто заявляет о незнании вычислений…

и мгновенный ток для тех, кто не боится вычислений…

I = к = дк
т дт

Единица измерения тока — ампер [А], названная в честь французского ученого Андре-Мари Ампера (1775–1836).В письменных языках без диакритических букв (а именно в английском) принято писать единицу измерения как ампер, , а в неформальном общении сокращать это слово до amp . У меня нет проблем с любым из этих вариантов написания. Только не используйте заглавную букву «А» в начале. Ампер относится к физику, а ампер (или ампер, или ампер) относится к единице.

Поскольку заряд измеряется в кулонах, а время измеряется в секундах, ампер равен кулону в секунду.



А = С

с

Элементарный заряд определяется как ровно…

e = 1,602176634 × 10 −19 C

Число элементарных зарядов в кулонах будет обратной величиной этого числа — повторяющейся десятичной дробью с периодом 778 716 цифр. Я напишу первые 19 цифр, это максимум, что я могу написать (поскольку произвольных долей элементарного заряда не существует).

C ≈ 6,241,509,074,460,762,607 e

А потом напишу еще раз с более разумным количеством цифр, чтобы было легче читать.

C ≈ 6,2415 × 10 18 e

Ток в один ампер — это передача примерно 6,2415 × 10 18 элементарных зарядов в секунду. Для любителей случайностей это примерно десять микромолей.

плотность тока

Когда я визуализирую ток, я вижу, как что-то движется.Я вижу, как они движутся в каком-то направлении. Я вижу вектор. Я вижу не то. Ток не является векторной величиной, несмотря на мою хорошо развитую научную интуицию. Ток — это скаляр. И причина в том … потому что это так.

Но подождите, становится еще страннее. Отношение силы тока к площади для данной поверхности называется плотностью тока.

Единица измерения плотности тока — ампер на квадратный метр , не имеющая специального названия.



A = A

м 2 м 2

Несмотря на отношение двух скалярных величин, плотность тока является вектором.И причина в том, что это так.

Ну… на самом деле, это потому, что плотность тока определяется как произведение плотности заряда и скорости для любого места в космосе…

Дж = ρ v

Два уравнения эквивалентны по величине, как показано ниже.

Дж = ρ v
Дж = q DS = с дк = 1 Я
В дт SA дт А
Дж = Я
А

Есть еще кое-что, что нужно учесть.

I = JA = ρ v A

Читатели, знакомые с механикой жидкостей, могли бы узнать правую часть этого уравнения, если бы оно было написано немного иначе.

I = ρ Av

Это произведение является величиной, которая остается постоянной в уравнении неразрывности массы .

ρ 1 A 1 v 1 = ρ 2 A 2 v 2

Точно такое же выражение применяется к электрическому току с символом ρ, меняющим значение между контекстами.В механике жидкости ρ обозначает массовую плотность, а в электрическом токе — плотность заряда.

микроскопическое описание

Ток — это поток заряженных частиц. Это дискретные сущности, а значит, их можно сосчитать.

n = N / V

q = нкВ

V = Ad = Av т

I = к = nqAv т
т т

I = nqAv

Аналогичное выражение можно записать для плотности тока.Вывод начинается в скалярной форме, но в конечном выражении используются векторы.

Дж = нк v

твердых частиц

Сравнение проводимости и валентных электронов, проводников и изоляторов

Дрейфовое движение, наложенное на тепловое движение

Увеличить

Мостовой текст.

Тепловая скорость электронов в проводе довольно высока и случайным образом изменяется из-за столкновений атомов.Поскольку изменения хаотичны, средняя скорость равна нулю.

Когда провод помещается в электрическое поле, свободные электроны равномерно ускоряются в промежутках между столкновениями. Эти периоды ускорения поднимают среднюю скорость выше нуля. (Эффект на этой диаграмме сильно преувеличен.)

тепловая скорость электрона в меди при комнатной температуре (классическое приближение)…

v rms = √ 3 (1.38 × 10 −23 Дж / К) (300 К)
(9,11 × 10 −31 кг)
v rms 100 км / с

ферми-скорость электрона в меди (квантовая величина)…

v fermi = √ 2 E Ферми
м e
v fermi = √ 2 (7.00 эВ) (1,60 × 10 −19 Дж / эВ)
(9,11 × 10 −31 кг)
v fermi 1500 км / с

Скорость дрейфа электрона на 10 м медного провода, подключенного к автомобильному аккумулятору 12 В при комнатной температуре (среднее время свободного пробега между столкновениями при комнатной температуре τ = 3 × 10 −14 с)…

v смещение = 1 v = 1 а τ = 1 Факс τ = 1 eE τ
2 2 2 м e 2 м e
v смещение = (1.60 × 10 −19 C) (12 В) (3 × 10 −14 с)
2 (10 м) (9,11 × 10 −31 кг)
v смещение 3 мм / с

Тепловая скорость на несколько порядков превышает скорость дрейфа в типичной проволоке. Время на прохождение круга — около часа.

жидкости

ионы, электролиты

газы

ионов, плазма

  • 14:02 — Линии электропередачи разъединяются на юго-западе Огайо
    4. Стюарт — Атланта 345 кВ
    Эта линия является частью пути передачи из юго-западного Огайо в северный Огайо. Он отключился от системы из-за возгорания кисти под частью линии. Горячие газы от пожара могут ионизировать воздух над линией электропередачи, заставляя воздух проводить электричество и закорачивать проводники.
    Источник

исторический

Символ I был выбран французским физиком и математиком Андре-Мари Ампера для обозначения силы тока силы тока.

Увеличить
Pour exprimer en nombre l’intensité d’un courant quelconque, on Concevra qu’on ait choisi un autre courant арбитраж для сравнения сроков…. Désignant donc par i et i раппортов интенсивных деяний двух партнеров, не имеющих отношения к объединению…. Чтобы выразить интенсивность тока в виде числа, предположим, что для сравнения выбран другой произвольный ток…. Воспользуемся я и я для соотношения интенсивностей двух заданных токов интенсивности опорного тока, принятым в качестве единицы ….
Андре-Мари Ампер, 1826 Андре-Мари Ампер, 1826 г. (платная ссылка)

Термин «интенсивность» теперь не имеет никакого отношения к физике. Ток — это скорость, с которой заряд протекает через поверхность любого размера — например, клеммы батареи или штыри электрической вилки. Интенсивность — это средняя мощность, на единицу площади, передаваемая некоторыми лучистыми явлениями, такими как звук оживленной дороги, свет Солнца или частицы брызг, испускаемые радиоактивным источником.Ток и интенсивность теперь — разные величины с разными единицами измерения и разным использованием, поэтому (конечно) они используют одинаковые символы.

текущий интенсивность
I = к

А = С

т с
I = P

Вт

А м 2

Начало стола

  • 12 000 А ток через магниты LHC в ЦЕРН

Что такое электрический ток? Определение, единица измерения и направление тока

Определение : Электрический ток определяется как скорость протекания отрицательных зарядов проводника.Другими словами, непрерывный поток электронов в электрической цепи называется электрическим током. Проводящий материал состоит из большого количества свободных электронов, которые беспорядочно перемещаются от одного атома к другому.

Единица тока

Поскольку заряд измеряется в кулонах, а время — в секундах, единицей измерения электрического тока является кулон / сек ( C / s, ) или амперы ( A ). Амперы — это единица измерения SI проводника. I — символическое представление тока.

Таким образом, считается, что по проводу проходит ток в один ампер, когда по нему течет заряд со скоростью один кулон в секунду.

Когда к металлическому проводу прикладывается разность электрических потенциалов, свободно прикрепленные свободные электроны начинают двигаться к положительному выводу ячейки, показанной на рисунке ниже. Этот непрерывный поток электронов составляет электрический ток. Токи в проводе протекают от отрицательного вывода ячейки к положительному выводу через внешнюю цепь.

Условное направление тока

Согласно теории электронов, когда к проводнику прикладывается разность потенциалов, через цепь протекает какое-то вещество, которое составляет электрический ток. Считалось, что это вещество течет от более высокого потенциала к более низкому потенциалу, т.е. положительный вывод к отрицательному выводу ячейки через внешнюю цепь.

Это соглашение о протекании тока настолько твердо установлено, что оно все еще используется.Таким образом, обычное направление потока тока — от положительного вывода элемента к отрицательному выводу элемента через внешнюю цепь. Величина протекания тока на любом участке проводника — это скорость потока электронов, то есть заряда, протекающего в секунду.

Математически это представлено как

В зависимости от протекания электрического заряда ток в основном подразделяется на два типа: переменный ток и постоянный ток. При постоянном токе заряды протекают в одном направлении, тогда как при переменном токе заряды протекают в обоих направлениях.

Q = It E = расчеты QV Закон Ома V = IR исследующие факторы, влияющие на сопротивление Характеристики графика ВАХ диод омический проводник лампа накаливания igcse / gcse 9-1 Physics revision notes

Электричество 3: Закон Ома, экспериментальные исследования сопротивления

и I-V графики и расчеты с использованием I = V / R, Q = It и E = QV

Редакция Доктора Брауна по физике Банкноты

Подходит для курсов GCSE / IGCSE Physics / Science или их эквивалент

Что такое закон Ома? Как вы делаете расчеты по закону Ома / с?

Какие факторы влияют на стойкость схема?

Как построить и использовать схему для исследовать закон Ома?

Как рассчитать количество электричества? заряд движется по цепи?


Субиндекс этой страницы

1.Закон Ома, простая схема исследования и расчеты V = IR

2. Движение и единица заряда, кулон, расчеты с использованием Q = It

3. Возможная разница и передача электроэнергии, E = QV расчеты

4а. Электрическое сопротивление — задействованные факторы

4б. Изучение сопротивление провода при постоянной температуре, различной длины и ширины

4с.Изучение вольт-амперные характеристики провода — объяснение графика

4д. Расследование ВАХ металлической лампы накаливания — график

4e. Изучение вольт-амперные характеристики диода — объяснение графика

См. Также ПРИЛОЖЕНИЕ 1 для обзора всей электроэнергии уравнения вам могут понадобиться


1а.Ома Закон (и упоминание других единиц, рассматриваемых в других разделах)

Закон Ома гласит, что ток через провод между двумя точками прямо пропорционален напряжению на две точки в цепи.

Он включает в себя самое фундаментальное уравнение что нужно знать для расчета электричества.

Математически это можно выразить как: я = V / R

перестановок: В = ИК и R = V / I

I = ток в амперах, амперах, А ; мера скорости потока электрического заряда.

В = разность потенциалов, п.о., вольт, В ; мера потенциальной энергии, передаваемой электрическому заряду течет.

Разность потенциалов в цепи это энергия , передаваемая на кулон электрического заряда , что течет между двумя точками в электрической цепи.

Кулон ( C ) — это единица электрического заряда (см. Q = Примечания к уравнению).

Передаваемая энергия рассчитывается от п.о. и количество электрического заряда ( Q ) перемещен p.d. в В (см. E = QV примечания к уравнениям).

R = сопротивление провода, Ом, Ом ; мера сопротивления проводника препятствовать поток заряда.

Чем больше сопротивление резистор, тем больше он сопротивляется и замедляет ток электричества.

Закон Ома означает, что R в этом уравнении является константой, не зависящей от величины протекающего электрического тока.

Закон правильно применяется к так называемому омическому сопротивлению . проводники , где протекающий ток прямо пропорционален приложенная разность потенциалов, но некоторые резисторы не подчиняются этому закону, например нагретая нить лампочка.

1b.Простой эксперимент по измерению сопротивления отдельного компонента

Если вы настроили контур 31 (правая диаграмма), вы можете измерить сопротивление постоянного резистора [R].

Изменяя напряжение источника питания с помощью переменной резистора, вы легко можете получить множество пар показаний p.d. (V) и текущее (А).

Затем используйте уравнение закона Ома (R = V / I), чтобы вычислить значение фиксированное сопротивление.

Затем вы можете усреднить значения R, рассчитанные для более надежный результат.

Более подробная информация приведена ниже, чтобы получить полную ВАХ графики, а также как получить сопротивление графическим методом.

Это основная установка для исследования вольт-амперные характеристики любого компонента R.

1c. Примеры расчеты с использованием Ома Закон V = IR

Q1 Когда стр.d. от 4,5 В приложен к сопротивлению, течет ток 0,5 А.

Какое значение имеет резистор?

R = V / I = 4,5 / 0,5 = 9,0 Ом

Q2 Сопротивление имеет значение 50 Ом.

Какой п.о. должен быть применен к нему чтобы через него протекал ток 5,0 А?

В = ИК = 5 x 50 = 250 В

3 квартал А п.d. 240 В подается на резистор нагревательного элемента сопротивлением 30 Ом.

Сколько тока проходит через обогреватель?

I = V / R = 240/30 = 8.0 А

4 квартал Три батареи на 1,5 В были подключены последовательно к трем лампочкам.

Если амперметр измеряет ток 0,50 А, какое сопротивление у каждой лампочки?

I = V / R, поэтому R = V / I = (3 x 1.5) / 0,50 = 9,0 Ом

Так как общее сопротивление = сумма сопротивления, сопротивление каждой лампочки = 9,0 / 3 = 3.0 Ом

5 квартал


ВЕРХ СТРАНИЦЫ и субиндекс


2. Движение заряда

2а. Расчет заряда, проходящего через точку в цепи Q = It

Ток (I в амперах) — это скорость протекания электрического заряжать вокруг цепи.

Чем больше поток заряда в данный момент времени, тем больше электрический ток.

Скорость протекания электрического заряда равна измеряется в кулонах в секунду.

Вы можете рассчитать заряд, проходящий через точку в цепи в заданное время по формуле

Q = Это

, где Q = электрический заряд в кулонах ( C ) — блок электрического заряда

I = ток в амперах ( A ) и t = время ( с )

перестановок из Q = It, I = Q / т и t = Q / I

Ток в 1 А равен скорость потока заряда 1 Кл / с.

2b. Примеры расчетных вопросов с участием уравнение Q = It

Q1 Если ток 3,0 А проходит через прибор в течение 1 часа 30 минут, сколько электрического заряда передается в процессе?

Q = It, Q = 3,0 x 1,5 x 60 x 60 = 16 200 C = 1,62 х 10 4 С

Q2 Если Заряд 9000 C проходит через точку в электрической цепи в 12.0 минут, какой текущий поток?

I = Q / t = 9000 / (12 x 60) = 9000/720 = 12,5 А

3 квартал Сколько времени займет электрическая цепь в минутах и ​​секундах ток 20.0 А передать 5000 С заряда?

t = Q / I = 5000/20 = 250 секунд = 4 минут и 10 секунд

Q4 Портативный компьютер зарядное устройство пропускает ток 1.20 А на 30 минут с выходом p.d. 15.0 В.

(а) Рассчитайте, сколько заряда перенесена в компьютерную батарею.

Q = It = 1,2 х 30 х 60 = 2160 С

(б) Какое сопротивление зарядное устройство?

В = ИК, R = V / I = 15 / 1,2 знак равно 12,5 Ом

(c) Когда аккумулятор ноутбука полностью заряжено в нем хранит 3000 с.

Сколько времени потребуется, чтобы полностью заряжать разряженный аккумулятор?

Q = It, t = Q / I = 3000 / 1,2 = 2500 с (41 мин 40 с)

5 квартал


ВЕРХ СТРАНИЦЫ и субиндекс


3. Возможная разница и передача энергии

3а.Введение в электрическую энергию перевод

Передача энергии на единицу заряда = разность потенциалов (п.о.) и расчеты на основе E = QV

В предыдущем разделе мы рассмотрели, как рассчитать количество заряда, движущегося в цепи, но ничего не сказал об энергии переведен.

Напоминания :

Электрические цепи, используемые термины, условные обозначения цепей, параллельные объяснение цепей и последовательных цепей

Разница потенциалов (стр.d. в вольт, В ) — энергия, передаваемая на единицу заряда в виде электрического заряд перемещается из одной точки в другую в электрической цепи.

Измеряется вольтметром, который всегда подключается параллельно компоненту схемы.

Передача электрического тока энергия

Подумайте обо всех электрических бытовые приборы, которыми вы пользуетесь — все они нуждаются в энергии для работы!

Блок питания работает на заряжается и передает ему энергию.

Работа должна производиться на зарядке для увеличения его потенциальной энергии.

Электрический заряд измеряется в кулоны (К)

Заряд и его движение уже было рассмотрено в раздел 2 (Q = It).

Заряды передают энергию компонентов по мере их прохождения, выполняя работу против сопротивления компонент.

Если работа сделана, то энергия переведен.

Если электрический заряд испытывает разность потенциалов, этот заряд будет течь, передавая энергию.

Энергия подается из энергоаккумулятора источник питания — аккумулятор, сеть и т. Д.

При прохождении заряда по любой п.d. падение высвобождает энергию (с более высокого уровня потенциальной энергии на более низкий).

например в тонкой проволоке сопротивление , выделяется тепло.

Разница потенциалов между двумя баллов равняется проделанной работе на единицу заряда.

разность потенциалов (В) = проделанная работа ( энергия передается в Дж) заряд (C)

т.е. 1 вольт соответствует 1 джоуля за кулон или V = J / C

Чем больше падение п.д., тем больше энергия передан, потому что заряд начинается с большим потенциалом энергия.

Следовательно, блок питания с большим источником p.d. (В) может подавать на больше энергии в схему на единицу электрического заряда ( кулон, В).

Чем больше p.d., тем больше энергии такое же количество электрический заряд может нести.

3b.Еще одно уравнение для расчета электрического передача энергии

В количество переносимой энергии можно рассчитать по формуле:

переданная энергия = заряд x потенциал разница.

E = QV (поэтому Q = E / V и V = E / Q )

E = передаваемая энергия в джоулях ( Дж )

Q = количество электрический заряд в кулонах ( C )

В = разность потенциалов ( В )

отмечая, что: V = E / Q = энергия, передаваемая на единицу заряда (J / C)

Попутно и несколько напоминаний:

Чем больше энергии передается в данном время, тем больше мощность устройства или электрического прибора.

The p.d. V говорит вам, сколько энергия, передаваемая на каждую единицу электрического заряда,

так, V = E / Q , (шт. J / C), см. Расчеты E = QV ниже).

Текущий I говорит вам, сколько заряд проходит заданную точку в цепи за единицу времени (кулонов в секунду, К / с ).

Это означает, что оба p.d. В и текущий I влияет на скорость передачи энергии к прибору из накопителя электроэнергии в другие накопители энергии.

И несколько математических связей на основе раздела 2. Q = It, а здесь в разделе 3 E = QV

Из Q = It и E = QV, подставляя дает E = ItV,

так (i) E = IVt (I в A, t в с, В в вольтах)

Перестановка E = IVt дает IV = Е / т

Это соединяется с уравнения для мощности

(ii) Мощность = энергия переведено / время получено = E / t (Дж / с), и

( iii ) Мощность = ток x напряжение = P (Вт) = I (A) x V (В), P = IV

Из (ii) и (iii) E / t = IV, поэтому E = IVt , что является уравнением (i) !!!

3c.Расчет q на основе E = QV (иногда с участием других уравнений электричества)

Q1 Электродвигатель Модель автомобиля питается от аккумулятора 1,5 В.

Если через цепь двигателя в движущемся вагоне,

(а) сколько энергии передается?

E = QV = 120 x 1,5 = 180 Дж

(b) Опишите вероятный накопитель энергии меняется, когда машина работает.

Химическая потенциальная энергия запас батареи уменьшается и превращается в электрическую энергию.

Накопитель кинетической энергии автомобиль увеличивается из-за потери тепла от трения и звуковой энергии переданы в накопитель тепловой энергии окрестностей.

Q2 Какое количество заряд необходим для передачи 500 Дж энергии, если п.о.цепи 24,0 В?

E ​​= QV, Q = E / V = 500/24 ​​= 20,8 С (3 SF)

Q3 Какой потенциал разность требуется в цепи для передачи 2000 Дж энергии с заряд 50 кулонов?

E ​​= QV, V = E / Q = 2000/50 = 40 В

4 квартал А 12.Батарея 0 В пропускает через лампу ток 2,0 А в течение 5 минут.

(а) Рассчитайте, сколько заряда прошло через лампу.

Q = It = 2 x 5 x 60 = 600 С

(б) Рассчитайте, сколько электроэнергии был перенесен лампой.

Два пути:

(i) E = QV = 600 x 12 = 7200 Дж, самый простой, но вы можете рассчитать его, не зная Q, из:

(ii) E = IVt = 2 x 12 x 5 x 60 = 7200 Дж

5 квартал Устройство имеет мощность 1.5 кВт и работает от сети 230 В.

Если прибор используется в течение 15 минут, сколько заряда прошло по цепи?

1,5 кВт ≡ 1500 Вт ≡ 1500 Дж / с

Общая переданная энергия = мощность x время = 1500 x 15 x 60 = 1 350 000 Дж

E ​​= QV, поэтому Q = E / V = ​​1350 000/230 = 5870 C (3 SF)

Ответ можно рассчитать по другой маршрут

P = IV, I = P / V = ​​1500/230 = 6.522 А

Q = It = 6,522 x 15 x 60 = 5870 C (3 SF)

Q6

3d. Еще немного о разности потенциалов — действие двух последовательно соединенных резисторов

На схеме 41 показаны два подключенных резистора. последовательно.

Справа показано, что происходит с p.d. по часовой стрелке по цепи (направление условного тока).

Повышается потенциальный запас аккумулятора разность зарядных потенциалов заряда до 12 В.

По мере прохождения заряда через 1-й резистор R 1 , он теряет энергию и п.д. падает на 8 В до п.п. из 4 В.

По мере прохождения заряда через 2-ю резистор R 1 , он снова теряет энергию и п.о. падает на 4 В до p.d. из 0 В.

Пока есть полная цепь, процесс повторяется.

Так как E = QV, выделяется вдвое больше энергии. резистором R 1 (p.d. 8 В), чем R 2 (p.d. 4 В) для тот же ток.


ВЕРХ СТРАНИЦЫ и субиндекс


4. Электрическое сопротивление — эксперименты по исследовать ВАХ различных сопротивлений и достоверность, или в противном случае Закона Ома

4а. Что влияет на сопротивление провода? Сопротивление постоянный?

и s графики вольт-амперных характеристик (ВАХ) объяснил

Сопротивление цепи зависит от несколько факторов:

(i) толщина сопротивления провод — для однокомпонентного резистора

(ii) длина сопротивления провод — для однокомпонентного резистора

(iii) если более одного сопротивления, они подключены последовательно или параллельно?

(iv) температура компонента действует как сопротивление

Раздел 4.описывает и объясняет несколько примеров графиков ВАХ — которые можно исследовать с помощью схемы 31 (справа)

На принципиальной схеме 31 справа показано как можно исследовать изменение тока через сопротивление (или любой компонент) при изменении разности потенциалов.

Графики разности потенциалов тока используются, чтобы показать, как ток через компонент изменяется в зависимости от разности потенциалов на нем.

Сопротивление некоторых резисторов / компонентов действительно меняется. как действующий и п.д. изменения например диодная или лампа накаливания.

Узнайте, как и почему в разделах 4d. и 4e.


ВЕРХ СТРАНИЦЫ и субиндекс


4b. Исследование электрического сопротивления провода — изменение длины или ширины

Схема 30 показывает, как исследовать сопротивление провода

Относительно тонкая проволока закреплена на каждый конец на линейке с разметкой в ​​миллиметрах с помощью зажимов типа «крокодил».

Вам понадобится амперметр для измерения ток в амперах и вольтметр для измерения p.d. через провод в вольт.

Провод подключается в серии с аккумуляторным блоком питания, переключателем и амперметром для измерения силы тока течет по проводу в амперах.

Вольтметр , для измерения p.d, есть подключен параллельно через резистивный провод.

Обратите внимание, что амперметр всегда подключается к последовательно с компонентом , но вольтметр всегда подключается параллельно по любому исследуемому компоненту.

Один конец провода, подключенный через вольтметр закреплен (слева), но на другом конце есть зажим типа «крокодил», который действует как подвижный точку контакта для размещения на различном расстоянии вдоль провода сопротивления от слева направо.

Замкните выключатель, чтобы замкнуть цепь. и начать снимать показания.

Лучше всего открыть переключатель между показания, чтобы свести к минимуму риск нагрева провода.

Вы можете изменять расстояние d (мм) от слева (0 мм) в точку дальше вправо и возьмите серию пар из п.d и текущие показания, например каждые 50 мм (можно работать в см, получается нет разницы!).

Используя закон Ома, вы вычисляете сопротивление в омах из уравнения R = V / I

Ты затем можно построить график зависимости сопротивления (Ом) от длины провода d (мм) — показано справа.

Вы должны обнаружить, что график является линейным с его начало координат x, y в точке 0,0.

Это означает, что сопротивление пропорционально длине провода .

Если не закрепить провод точно на 0 мм, график по-прежнему должен быть линейным, но начало линии не будет быть 0,0.

Если повторить эксперимент с провода разного диаметра, вы должны обнаружить, что градиент становится ниже, толще проволока.

Для провода той же длины сопротивление меньше толще проволока — хорошая аналогия — легкость, с которой вода течет через труба тонкого или большего диаметра.


ВЕРХ СТРАНИЦЫ и субиндекс


4c. Изучение ток — напряжение характеристики провода

Это эксперимент по закону Ома

Схема 31 показывает, как исследовать зависимость I от V для сопротивления

.

Следствие сосредоточено на поиске из …

… как протекает ток через резистор меняются в зависимости от разности потенциалов на нем?

Постоянный резистор представляет собой ‘компонент’ в цепи и должен иметь постоянную температуру на протяжении всего эксперимента (см. ниже температура последствия).

В этом случае простой проволочный резистор соединены последовательно с блоком питания и амперметром.

The p.d. измеряется по фиксированному сопротивление с вольтметром,

Тем не мение, также подключенный последовательно, добавлен переменный резистор, так что вы можете удобно изменить разность потенциалов и тем самым изменить ток протекает через компонент.

Это позволяет собрать целую серию пар показаний I и V, с помощью которых можно построить подходящие графики — в этом случай V против I, но часто делается как I против V.

Используя переменный резистор, вы постепенно увеличивайте разность потенциалов на компоненте, принимая соответствие текущему чтению, например увеличивается на 0,5 В. за раз. Повторите каждый читать дважды и использовать среднее значение.

После этого можно поменять местами клеммы аккумулятора. и повторить все показания.

Если вы построите p.d. по сравнению с текущим, график линейный , если он подчиняется закону Ома — тогда он называется ‘ омический кондуктор ‘.

Это я изобразил графиком вверху справа, а градиент равен сопротивлению в Ом .

Это соответствует закону Ома уравнение V = IR , поэтому градиент равен R .

Если график остается линейным, сопротивление остается постоянным.

Этот график не представляет показания сняты после перестановки полюсов аккумулятора.

Однако показывает, как получить значение сопротивления графическим методом.

Это линейный график и фраза линейный компонент может использоваться.

Для таких компонентов, как провод, который не нагревается, вы должны получить линейный график p.d. (V) против I (A) с градиент R (Ω). (правый график).

Вы должны убедиться, что провод не сильно нагревается — если начинает нагреваться, сразу отключите резистор («выключите») и дайте ему остыть.

Если вы построите график зависимости I от V, градиент будет 1 / R (обратная величина сопротивление), линейный график .

Этот график показывает, что вы получаете построение всех данных, включая показания I-V, снятые после реверсирования клеммы аккумулятора.

График (1) является построенный на поперечной оси. Верхняя правая половина — это ваш первый набор результатов, затем вы переверните клеммы на источнике питания и повторите эксперимент дает нижнюю левую часть графика.

Обратите внимание, что вы получите только линейный график, если температура проволоки остается постоянной .

Когда ток (А) пропорционален к p.d (V) он описывается как омический провод (подчиняется закону Ома!).

С помощью схемы 31 вы можете проверить любой резистор или любой другой тип компонента схемы, и результаты обсуждается ниже, начиная с резюме факторов, влияющих на сопротивление.

Так, сопротивление омического проводника, например цепь компонент не меняется независимо от того, проходит ли ток — постоянный градиент 1 / R для графика 1.

Это ожидаемые линейные графики для фиксированный резистор с использованием схемы 31 выше.

Думая против часовой стрелки на диаграмме, разные линии графика могут отображать убывающее сопротивление е.грамм. (i) резистивный провод становится короче при том же диаметре, или (ii) увеличение диаметра при фиксированной длине провода.

При постоянной температуре ток, протекающий через омический проводник прямо пропорционален разности потенциалов на это — уравнение: В = IR или I = V / R .

Однако это только правда, так как линейный график, если температура не изменять.

Комментарии к переменным в этом частный эксперимент по закону Ома

Ток всегда определяется сочетание п.о. (В) и сопротивление R (Ом).

Независимая переменная что мы изменяем или контролируем в эксперименте — в этом случае вы можете считать его р.д. управляется переменным резистором.

Одно из соглашений состоит в том, чтобы построить график независимая переменная на оси x, а зависимая переменная на оси ось y.

Этот означает, что сопротивление R, является обратной величиной градиента — немного больше неудобно рассчитывать сопротивление, чем на графике V в зависимости от I, где градиент — это сопротивление. Закон Ома: I = В / р.

Зависимая переменная — это то, что мы тестируем или измеряем в эксперименте, это ток I (A), который зависит от настройки переменного резистора, который, в свою очередь, контролирует разность потенциалов на резисторе.

Управляющие переменные — это то, что мы сохраняем то же самое во время эксперимента, чтобы убедиться, что это честный тест например в этом случае сохраняется провод и температура постоянная, НЕ должна изменяться — не меняйте провод и не допускайте нагреть.


ВЕРХ СТРАНИЦЫ и субиндекс


4г. Исследование вольт-амперных характеристик металла лампа накаливания

Когда электрический заряд проходит через высокое сопротивление, как тонкая металлическая нить лампы накаливания, она передает часть электроэнергия к накопителю тепловой энергии нити накала. Электрический заряд работают против сопротивления .

Схема 45 показывает, как можно исследовать характеристики разности токов и потенциалов нить лампочка.

Вольтметр подключен параллельно термистор, п.д. В измеряется в вольтах ( В ).

Переменный резистор позволяет варьировать п.д. и текущий поток.

Амперметр, подключенный последовательно, дает текущее значение I в амперах ( A, ).

В прохождение тока нагревает нить накала и поднимается в температура вызывает повышение сопротивления . Итак, нить Лампа — это неомический провод .

Этот «эффект нагрева» влияет на все резисторы.

По мере увеличения тока выделяется больше тепловой энергии, и нить накаливания становится все горячее и горячее, поэтому дальнейшее повышение температуры еще больше увеличивает сопротивление.

Это уменьшает скорость увеличения тока с увеличение разности потенциалов .

Следовательно, градиент I-V кривая графика уменьшается, и все больше с увеличением температура — график 2. Это нелинейный график .

Если градиент меняется, тогда сопротивление меняется.

График (2) является построенный на поперечной оси.Верхняя правая половина — это ваш первый набор результатов, затем вы переверните клеммы на источнике питания и повторите эксперимент дает нижнюю левую часть графика.

Фраза нелинейная компонент может использоваться.

Когда ток (А) НЕ пропорциональная p.d (V) лампа накаливания описывается как неомический провод (не подчиняется закону Ома!).

У вас такой же график в форме треугольника. для термистор.

Теория — со ссылкой на схему металлической конструкции

Металлическая кристаллическая решетка состоит из неподвижных ионов и свободно движущихся электроны между ними. С повышением температуры ионы металла колеблются сильнее. сильно, в котором электроны сталкиваются, и это препятствует прохождению электронов, уменьшая поток обвинять. По мере увеличения тока вибрации увеличиваются, вызывая больше электрической энергии, которая будет преобразована в тепло — увеличивая температура И сопротивление металлической нити, тем самым еще больше понижая ток.

Итак, увеличиваем i n по температуре увеличивает сопротивление лампы накаливания (или большинства других резисторы) и снижает ток, протекающий для заданного p.d.

Если резистор становится слишком горячим, ток почти не будет течь.

Есть одно важное исключение к этому «правилу», см. примечания к термистор, где сопротивление фактически падает с повышение температуры.

Лампа накаливания — лишь одна из многих примеры были энергия передается с пользой , НО есть всегда теряется тепловая энергия в накопитель тепловой энергии устройства и Окрестности .

Нить накала часто изготавливается из металлический вольфрам, плавящийся при> 3400 o C, и ярко светящийся при 2500 o C, но все равно испаряется очень медленно. Инертный газ например, добавляется аргон или азот, чтобы уменьшить это испарение — любое испаренные атомы вольфрама попадают в инертный (и, следовательно, неокисляющий) Ar или N 2 молекул и, надо надеяться, снова конденсируются на нити.

См. Сохранение энергии, передача-преобразование энергии, эффективность — расчеты


ВЕРХ СТРАНИЦЫ и субиндекс


4e. Исследование вольт-амперных характеристик диода

Модель ток через диод протекает только в одном направлении — см. график 3.

Сопротивление в обратном направлении очень высока — следовательно, это фактически «односторонняя» система.

Схема 43 показывает, как можно исследовать вольт-амперная характеристика диода.

Вольтметр подключен параллельно термистор, п.д. В измеряется в вольтах ( В ).

Переменный резистор позволяет варьировать п.д. и текущий поток.

Амперметр, подключенный последовательно, дает текущее значение I в амперах ( A, ).

Диод имеет очень высокое сопротивление в обратное направление .

Также есть порог р.о. (например, 1,4 В) до того, как ток вообще потечет — внимательно посмотрите на график — есть короткий горизонтальный участок, прежде чем ток поднимется с нуля и со временем становится линейным.

Таким образом, вы получаете верхнюю правую часть графика 3 по сравнению с графиками 1 и 2 выше.

Это потому, что когда вы делаете экспериментируйте, используя схему, описанную выше, по изменению подключений, вы не обнаружите никаких текущих потоков при изменении p.d.

Его нелинейный График .

Если градиент меняется, то сопротивление меняется.

Когда ток (А) НЕ пропорциональный p.d (V), диод описывается как неомический провод (не подчиняется закону Ома!).

Фраза Нелинейный компонент может быть использовано.

График (3) является построенный на поперечной оси. Верхняя правая половина — это ваш первый набор результатов, затем вы переверните клеммы на источнике питания и повторите эксперимент дает нижнюю левую часть графика.

Так как ток течет только в одну сторону через диод его можно использовать для преобразования переменного тока в постоянный Текущий.


ВЕРХ СТРАНИЦЫ и субиндекс


Практическая работа по помощь в развитии ваших навыков и понимания, возможно, включала следующее:

с использованием ламп накаливания и резисторов для исследования разности потенциалов — Текущий характеристики,


ПРИЛОЖЕНИЕ 1: Важные определения, описания, формулы и ед.

Примечание: Вы можете / можете нет (но не волнуйтесь!), столкнулись со всеми этими терминами, это зависит от как далеко продвинулась ваша учеба.В вашем курсе вам может не понадобиться каждая формула — решать вам.

V разность потенциалов ( p.d ., обычно называемая « напряжение ») — это движущий потенциал, который перемещает электрический заряд вокруг схема — обычно электронов .

Возможная разница — это работа, выполненная в перемещение единицы заряда.

Показывает, сколько энергии передается за единицу заряда, когда заряд перемещается между двумя точками в цепи е.грамм. между выводами аккумуляторной батареи.

г. в любой части цепи измеряется в вольтах, В .

Я ток — это скорость протекания электрического заряда в кулонов в секунду ( C / s ), измеряется в амперах (амперах, A, ).

Количество переданного электрического заряда a give time = текущий расход в амперах x прошедшее время в секундах

Формула соединения: Q = Это , I = Q / t, t = Q / I, Q = электрический заряд перемещается в кулонов ( C ), время т ( с )

рэнд сопротивление в цепи, измеренное в Ом ( Ом ).

Сопротивление замедляет прохождение электрического заряда — он противостоит потоку электрического заряда .

Формула соединения: В = ИК , I = V / R, R = V / I (Это формула для Закон Ома)

P является мощность , передаваемая цепью = показатель энергии передача ( Дж / с, ) и измеряется в Вт, ( Вт, ).

Формула соединения: P = IV , I = P / V, V = P / I также P = I 2 R (см. также P = E / t ниже)

E = QV , энергия, передаваемая количеством электрического заряда потенциалом разность вольт.

переданной энергии (джоулей) = количество электрического заряда (кулоны) x разность потенциалов (вольт)

Q = E / V, V = E / Q, E = передача энергии в джоулях ( J ), Q = электрический заряд перемещен ( C ), V = p.d. ( В )

E = Pt , P = E / t, t = E / P, где P = мощность ( Вт, ), E = переданная энергия ( Дж) , t = затраченное время ( с )

Передаваемая энергия в джоулях = мощность в ваттах. x время в секундах

Формула связи: Поскольку E = Pt и P = IV, передаваемая энергия E = IVt


ВЕРХ СТРАНИЦЫ и субиндекс


Что дальше?

Электричество и ревизия магнетизма индекс нот

1.Полезность электроэнергии, безопасность, передача энергии, расчеты стоимости и мощности, P = IV = I 2 R, E = Pt, E = IVt

2. Электрические схемы и как их рисовать, условные обозначения схем, параллельность схемы, объяснение последовательных схем

3. Закон Ома, экспериментальные исследования сопротивление, I-V графики, расчеты V = IR, Q = It, E = QV

4. Схемы устройств и как они используются? (е.грамм. термистор и LDR), соответствующие графики gcse Physical Revision

5. Подробнее о последовательных и параллельных цепях. электрические схемы, измерения и расчеты gcse физика

6. Электроснабжение «Национальной сети», экология вопросы, использование трансформаторов gcse примечания к редакции физики

7. Сравнение способов получения электроэнергии gcse Заметки о пересмотре физики (энергия 6)

8.Статическое электричество и электрические поля, использование и опасность статического электричества gcse примечания к редакции физики

9. Магнетизм — магнитные материалы — временные (индуцированные) и постоянные магниты — использует gcse физика

10. Электромагнетизм, соленоидные катушки, применение электромагнитов gcse примечания к редакции физики

11. Моторное воздействие электрического тока, электродвигатель, громкоговоритель, правило левой руки Флеминга, F = BIL

12.Эффект генератора, приложения напр. генераторы производство электричества и микрофон gcse физика

ВСЕ мои GCSE Примечания к редакции физики

ИЛИ воспользуйтесь [GOGGLE ПОИСК]



Версия IGCSE заметки по закону Ома исследования сопротивления V = IR Q = It вычисления KS4 физика Научные заметки на Исследование сопротивления по закону Ома V = IR Q = It расчеты GCSE руководство по физике примечания к закону Ома исследования сопротивления V = IR Q = It расчеты для школ, колледжей, академий, научных курсов, репетиторов изображений рисунки-диаграммы для исследования сопротивления по закону Ома V = IR Q = Он вычисляет научные исправления Исследование сопротивления по закону Ома V = IR Q = It расчеты для пересмотра модулей физики примечания по темам физики, чтобы помочь в понимании Исследование сопротивления по закону Ома V = IR Q = It расчеты университетские курсы физики карьера в науке и физике вакансии в машиностроении технический лаборант стажировка инженер стажировка по физике США 8 класс 9 класс 10 AQA Примечания к пересмотру GCSE 9-1 по физике по закону Ома исследования сопротивления V = IR Q = It вычисления GCSE примечания к закону Ома исследования сопротивления V = IR Q = It расчеты Edexcel GCSE 9-1 физика наука примечания к пересмотру Исследование сопротивления по закону Ома V = IR Q = It расчеты для OCR GCSE 9-1 21 век научные заметки по физике об исследованиях сопротивления по закону Ома V = IR Q = Расчет OCR GCSE 9-1 Шлюз физики примечания к пересмотру исследований сопротивления по закону Ома V = IR Q = It вычисления WJEC gcse science CCEA / CEA gcse science

ВЕРХ СТРАНИЦЫ и субиндекс

Какова сила электрического тока? Сила тока

Сила тока — это электрический заряд, который проходит через участок проводника за секунду.

Электрический ток в проводе, где носителями заряда являются отрицательно заряженные электроны, является мерой количества заряда, проходящего через любую точку провода за единицу времени.

Поток положительных зарядов (например, протонов или положительных ионов) имеет такой же эффект в цепи, как равный поток электронов в противоположном направлении.

Следующая формула дает силу тока:

I = Q / t

Где:

  • «I» представляет силу электрического тока, выраженную в амперах (A).

  • «Q» представляет собой электрический заряд, выраженный в кулонах (Кл).

  • «t» представляет время, выраженное в секундах.

С этим определением мы можем сказать, что электрический ток имеет силу 1 ампер, когда нагрузка в 1 коломбий проходит через участок проводника за 1 секунду.

Что такое единица измерения силы электрического тока?

Единицей измерения силы тока является ампер А в Международной системе единиц.

Ампер назван в честь Андре-Мари Ампера.

Сила электрического тока измеряется гальванометром. Для правильного измерения гальванометр должен быть включен последовательно с электрическим проводником, сила тока которого должна измеряться.

Амперметр — это гальванометр, калиброванный в амперах.

Что такое единицы ампер-часы?

Ампер-час — это единица измерения количества электрического заряда, протекающего через запоминающее устройство, если оно подает ток силой 1 ампер в течение 1 часа.Ампер-час равен 3600 кулонам.

Как генерируется сила тока?

Представьте себе два конца одного и того же проводника. Каждая партия подвергается воздействию различного электрического поля с разными электрическими потенциалами.

Из-за этой разности потенциалов электроны в проводнике испытывают силы притяжения и отталкивания. Эти электрические силы заставляют свободные электроны двигаться.

Электрические заряды движутся с определенной скоростью.

Добавить комментарий

Ваш адрес email не будет опубликован. Обязательные поля помечены *